Jump to content

Wikipedia:Reference desk/Science: Difference between revisions

From Wikipedia, the free encyclopedia
Content deleted Content added
→‎downcoast?: new section
Line 374: Line 374:


Does Oiling the head-hair, for heaving "wet-look", has any biological advantages or disadvantages? thanks. <span style="font-size: smaller;" class="autosigned">— Preceding [[Wikipedia:Signatures|unsigned]] comment added by [[Special:Contributions/79.179.151.250|79.179.151.250]] ([[User talk:79.179.151.250|talk]]) 19:06, 1 June 2012 (UTC)</span><!-- Template:Unsigned IP --> <!--Autosigned by SineBot-->
Does Oiling the head-hair, for heaving "wet-look", has any biological advantages or disadvantages? thanks. <span style="font-size: smaller;" class="autosigned">— Preceding [[Wikipedia:Signatures|unsigned]] comment added by [[Special:Contributions/79.179.151.250|79.179.151.250]] ([[User talk:79.179.151.250|talk]]) 19:06, 1 June 2012 (UTC)</span><!-- Template:Unsigned IP --> <!--Autosigned by SineBot-->

== downcoast? ==

how is the time called when a mashine shut down and the components rotating till they stop?
on sirens it is called coast down but how on machines?

Revision as of 19:48, 1 June 2012

Welcome to the science section
of the Wikipedia reference desk.
Select a section:
Want a faster answer?

Main page: Help searching Wikipedia

   

How can I get my question answered?

  • Select the section of the desk that best fits the general topic of your question (see the navigation column to the right).
  • Post your question to only one section, providing a short header that gives the topic of your question.
  • Type '~~~~' (that is, four tilde characters) at the end – this signs and dates your contribution so we know who wrote what and when.
  • Don't post personal contact information – it will be removed. Any answers will be provided here.
  • Please be as specific as possible, and include all relevant context – the usefulness of answers may depend on the context.
  • Note:
    • We don't answer (and may remove) questions that require medical diagnosis or legal advice.
    • We don't answer requests for opinions, predictions or debate.
    • We don't do your homework for you, though we'll help you past the stuck point.
    • We don't conduct original research or provide a free source of ideas, but we'll help you find information you need.



How do I answer a question?

Main page: Wikipedia:Reference desk/Guidelines

  • The best answers address the question directly, and back up facts with wikilinks and links to sources. Do not edit others' comments and do not give any medical or legal advice.
See also:


May 28

The article on Catenane says that "The interlocked rings cannot be separated without breaking the covalent bonds of the macrocycles", but no source is cited. Is this correct? As two covalent bonds come closer to crossing one another, couldn't the entangled electrons of one bond tunnel through the point of the other pair and hence break the chain without breaking either of the two macrocycles? 203.27.72.5 (talk) 03:37, 28 May 2012 (UTC)[reply]

Although tunnelling would be possible, it would be very very unlikely, as you would also have to move the whole of more than one atom and not just the electrons, so say instead of a one in a million chance it may be one in a zillion chance. However you may be interested in living cells there is Type II topoisomerase which can pass DNA through another strand. Graeme Bartlett (talk) 12:47, 28 May 2012 (UTC)[reply]
How unlikely? Like if you had a massive catanane 1km long wouldn't the chain breaking at some point become extemely likely as time passes? 1.124.213.55 (talk) 08:58, 1 June 2012 (UTC)[reply]

Comparing Heat Conductivities

In general, out of all the different materials that water bottles are made up of, which has the lowest heat conductivity. The other day, I saw a Voss bottle (which is cool because it is made up of glass). So I started thinking about is a plastic bottle better or glass. I want the water to stay cool as long as possible in ambient air temperature so that it doesn't warm up quickly. I understand that it would also make it tougher for the refrigerator to cool the water down (low heat conductivity works both ways, right?). So someone who knows more about the types of plastics and glass used to make consumer grade commercial water bottles, which kind is better? I don't use a thermos or anything. I just buy bottled water and then just use the bottle and replace it maybe every six months or so. Doesn't plastic also break down quicker (into carcinogenics) with heat acting as a catalyst? I remember my chemistry teacher vaguely saying something like this. - Looking for Wisdom and Insight! (talk) 05:45, 28 May 2012 (UTC)[reply]

I have a suspicion the glass bottle might feel cool to the touch because it conducts heat (away from your hand) better - which is the opposite of what you want. --Demiurge1000 (talk) 14:19, 28 May 2012 (UTC)[reply]

That is true that the glass bottle does feel much cooler but the water also feels cooler to me longer than it would from a plastic bottle. So the glass takes the heat in from its outside environment (because it is hotter) and then transfers that heat to the water more efficiently to heat it up? So a glass bottle warms up the water better until the inside and outside reach thermal equilibrium? Could it be just a weird perception error on my part? Like I touch the bottle, it feels very cool so I say to myself that the water is cool too while it may not really be? Or is there a scientific explanation for this and the glass bottle really does keep the water cooler for longer? I am thinking heat capacity, if the glass has high heat capacity then wouldn't it just absorb the heat from both sides and keep both sides cool? Cool to the touch and cool water? - Looking for Wisdom and Insight! (talk) 20:15, 28 May 2012 (UTC)[reply]

Yes, something like that. Also, a dark plastic bottle would absorb solar heat more than a clear glass bottle with a clear liquid inside.
And why not consider a thermos bottle ? It would keep your water cold and you won't get those nasty chemicals leaching from plastic. StuRat (talk) 04:00, 29 May 2012 (UTC)[reply]

Blind Spot in space travel

"Ships disappear in the Blind Spot. No puppeteer would go too near a singularity in hyperdrive; yet still they disappeared, in the days when our ships carried pilots. I trust the engineers who built the Liar. Hence I trust the cabin gravity. It will not fail us. But even the engineers fear the Blind Spot."

That's taken from page 103 of my copy of Ringworld. This Blind Spot is mentioned a few other times previously in the book and I didn't think too much of it because I figured some explanation of it would come. I don't think I've come across one though. Can anyone tell me what this Blind Spot would be? Thanks, Dismas|(talk) 10:42, 28 May 2012 (UTC)[reply]

It's a fictiony thing Niven invented; it's explained in Known Space. -- Finlay McWalterTalk 11:01, 28 May 2012 (UTC)[reply]
(EC) I'm pretty sure this is something that's primarily in-universe. The first and third internet search results for 'blind spot ringworld' for me, i.e. Known Space and [1] seem to answer the question although they may also contain spoilers (although I also wonder if the answer is a spoiler). Nil Einne (talk) 11:06, 28 May 2012 (UTC)[reply]
Not particularly spoilery, as I seem to recall that Niven explained it at its first introduction.
It's not a spacial location, despite the quote above, in which Niven is being metaphorical. Essentially, when one travels faster-than-light in Niven's version of hyperspace, one cannot properly visually perceive hyperspace itself, so if one's ship has a porthole or other transparent hull aperture (Puppeteer ships' hulls, which many other species use, can be made selectively or entirely transparent, according to convenience), it is as if a human's normal blind spot expands to cover the whole field of hyperspacial view, such that object either side of the porthole (say) appear to be next to each other with nothing between them perceptible. Additionally, however, many intelligent species (including Puppeteers and Humans) are psychologically drawn to stare at the hyperspacial Blind Spot, and individuals usually enter a kind of trance state in which thought and movement cease, which may persist until some mechanism or a still-un-tranced crewmate (where the ship is not a one-person craft) interrupts the view.
Hence, for a ship to have "disappeared in the Blind Spot", the pilot/crew has/have become hypnotised by it and ceased to navigate the ship, which will either crash or at least become permanently lost in hyperspace. {The poster formerly known as 87.81.230.195} 84.21.143.150 (talk) 12:15, 28 May 2012 (UTC)[reply]
It's thirty years since I read the stuff, but that sounds right to me. —Tamfang (talk) 21:07, 28 May 2012 (UTC)[reply]

Thanks for all that. I was hesitant to read too much about the book or the universe in it for fear of any spoilers. Dismas|(talk) 00:23, 29 May 2012 (UTC)[reply]

Algae on the Moon?

Timo Vuorensola, the director of Iron Sky says in this BBC article that algae was found on the moon. I cannot find anything on the internet about this, and it is the first time I've heard it. Surely, if it were true, it would be common knowledge, being a pretty sensational discovery. Is it true? Can anyone find any reliable reference about it? KägeTorä - (影虎) (TALK) 13:17, 28 May 2012 (UTC)[reply]

He's the director of a comedy film involving Nazis living on the dark side of the moon, so how could that qualify him as an expert on real lunar exploration? Roger (talk) 13:31, 28 May 2012 (UTC)[reply]
(EC) Certainly, I do not dispute that. However, for him to say something like that in a serious interview, I would expect there to be at least some information somewhere. I doubt he just made it up on the spot. Anyway, my question is asking for any links to information. I'm not here to provoke a discussion on the subject. KägeTorä - (影虎) (TALK) 13:40, 28 May 2012 (UTC)[reply]
If you read it again, you will see that he puts that comment into the context of one possibility, it is not stated as a fact. 217.158.236.14 (talk) 13:38, 28 May 2012 (UTC)[reply]
Sure, it is spoken in the context of a possibility, but the wording states it as fact: "Twenty years after the Nazi era, ice was found there, and algae". Perhaps he is confusing it with some stories I heard in the past about algae having 'possibly' been found on Mars, and not the Moon. KägeTorä - (影虎) (TALK) 13:43, 28 May 2012 (UTC)[reply]
I'd agree that is a possibilty. He is incorrectly remembering the widely reported discovery of supposed "micro fossils" in a Martian meteorite Allan Hills 84001. Roger (talk) 13:53, 28 May 2012 (UTC)[reply]

"New form of LSD"

Followers of the zombie genre have little difficulty explaining a recent face-eating cannibal attack in Miami. But police are quoted claiming it was due to "a new form of LSD". [2] Now of course there is only one lysergic acid diethylamide, and my understanding is hallucinogens of this class are generally credited with leaving an unaffected "watcher", some part of the brain which continues to monitor self-preservation and basic morality. (Henbane/scopolamine, on the other hand, who knows?) Anyway - anybody know what "new kind of LSD" that the police might be talking about? Wnt (talk) 21:55, 28 May 2012 (UTC)[reply]

It doesn't sound like that assessment is based on any deep understanding of chemistry — sounds like a policeman saying the equivalent of, "it's probably some crazy new drug I've heard about." "...their organs are burning up alive" is not a statement that reflects any attention to biological precision, either. Anyway, the "take your clothes off and do crazy, violent, even cannibalistic things" sounds like the "classic" symptoms of extreme PCP consumption, so I'm surprised it was not brought up in the report (or by the cops). --Mr.98 (talk) 22:07, 28 May 2012 (UTC)[reply]
Considering PCP isn't really in the news much any more, it may be PCP that the officer is referring to and he simply doesn't realize that it's an older drug. Dismas|(talk) 00:15, 29 May 2012 (UTC)[reply]
I suspect the officer may have said "PCP" rather than "LSD" and the reporter got it wrong, but that's just speculation. I doubt the officer doesn't know the difference — all law enforcement folks in big cities know that PCP is bad news and LSD is just annoying (from a police perspective). --Mr.98 (talk) 00:25, 29 May 2012 (UTC)[reply]
Chemically, there aren't really any "forms" of LSD. Either something is lysergic acid diethylamide or it isn't. The first impression I had upon hearing of the story (particularly of the fact that it took around four gunshots to drop the guy) is that PCP was involved. Else, very few people are going to be able to remain upright and physically active after the first gunshot, let alone the subsequent ones. Just speculation, though. Evanh2008 (talk|contribs) 03:55, 29 May 2012 (UTC)[reply]
The "bath salts" class of designer drugs has been in the news recently which may be what you're talking about. Whatever substance it is, it may be on the list at Designer drug, or could be something as old as a Mandrake (plant) derivative, which itself has a very interesting history. Shadowjams (talk) 16:28, 29 May 2012 (UTC)[reply]
It looks like the press is moving in that direction - though it is hard to connect the reports with any certainty. I suppose that the plan is to get some new felony involving methedrone, methylone, and methylenedioxypyrovalerone passed before the results from the forensic lab come back (See also Pamela's Law) Wnt (talk) 01:03, 30 May 2012 (UTC)[reply]
Well, some of what you named aren't innocuous substances, and some I believe are already illegal under the Analog Act (the designer drug article talks about it and links to the article). The Controlled Substances Act is at times alarmingly specific, and at other times alarmingly vague. I don't have a science background, but I read through the statute once and it's very interesting to see the specificity (I believe the DEA has rule-making power so a lot of the designer drugs that are made illegal are done so under the CFR... but I could be wrong about that... so what's in the U.S. Code is a small segment of the actual law). And of course the categorization of some controlled substances is quite arbitrary. Shadowjams (talk) 07:02, 30 May 2012 (UTC)[reply]
According to the DEA cathinones can produce similar symptoms to LSD. This seems very unlikely since they are stimulants, not psychedelics. 'Bath salts' appears to being used synonymously with designer drug in the US - i.e. an unidentified white powder. The only designer psychedelic I can think of is Bromo-DragonFLY. As you point out, there is no lab data yet, so anything is entirely speculative. Remember the Kony guy being found naked on the street? Sometimes the mind can do weird things without any chemical input. Considering the recent history of media reporting of mephedrone in the UK and the US (I've seen articles in very RS saying that people were snorting real bath salts to get high) I'm extremely skeptical to believe what they report. SmartSE (talk) 19:39, 31 May 2012 (UTC)[reply]


May 29

Central Sleep Apnea theory contested

The following discussion is closed. Please do not modify it. Subsequent comments should be made on the appropriate discussion page. No further edits should be made to this discussion.


Gentlemen

Some weeks ago for a second time I included in Wikipedia article on Sleep Apnea a short text titled "Central Sleep Apnea Contested". Both times this text was erased from Wikipedia the very first day in which it was posted.

On the other hand, HuffingtonPost published a post I sent on this same issue and they unposted it after about 15 minutes. But they sent my article to what they call their “Medical Preview” and after 15 days they got the medical approval of my revolutionary theory on this subject and they posted my Article: http://voces.huffingtonpost.com/../../antonio-garcia/para-superar-la-apnea-del_b_1501414.html.

Extensive text removed by Franamax. This is self-promotional / soapboxing unsuitable for the Reference Desk (no atucal question was asked) and should not appear in search engines. The removed text is available in page history. Franamax (talk) 02:55, 29 May 2012 (UTC)[reply]

— Preceding unsigned comment added by Genserico (talkcontribs) 00:04, 29 May 2012 (UTC)[reply]

The correct place to bring this up would be the sleep apnea talk page. However, removing your changes to the sleep apnea article was the correct thing to do. The main problem with your edits is that the book that you wrote does not count as a reliable source, in part because you are just a writer and a historian, not a doctor. I think it also counts as original research, since you yourself wrote the book that you're trying to promote in Wikipedia. And since you're self-promoting your own work, it also makes it hard for your edits to present a neutral point of view. Red Act (talk) 01:58, 29 May 2012 (UTC)[reply]
Note also that your mention of "legal consequences for Wikipedia" possibly violates Wikipedia's policy on legal threats. I recommend in the strongest possible terms that you delete that part of your statement. Short Brigade Harvester Boris (talk) 02:40, 29 May 2012 (UTC)[reply]
I've removed the bulk of the original post as it seems more intended to get onto a search engine than seek anything the RefDesks actually provide. The removed text can bee seen here. Franamax (talk) 02:57, 29 May 2012 (UTC)[reply]
The discussion above is closed. Please do not modify it. Subsequent comments should be made on the appropriate discussion page. No further edits should be made to this discussion.

Low frequency sound waves

Apparently low frequency sound waves can travel very far, but direction of the source of such a sound wave can be hard to determine. What's the physics behind this (I'd prefer a technical answer). Thanks! 65.92.6.118 (talk) 01:18, 29 May 2012 (UTC)[reply]

I think that high frequency sound-waves are informationally more dense than low frequency sound-waves, so they are more easily scrambled by the environment as the propagate. It's like writting with a fountain pen - if you write in a large font size, and acidentaly smudge it as you do, then it is easier to make out what it is supposd to say, compared with a small font size. Sound-waves decay as the propagate, high frequency sound-waves are just easier to merge with background noise. It also depends on the evironment, differet materials are prone to absorb different frequancies of sound-waves than others. Plasmic Physics (talk) 03:32, 29 May 2012 (UTC)[reply]
The further from the source sound wave has travelled the larger the uncertainty becomes. The same reason why the arc length increases with the radius. It's just basic maths. Plasmic Physics (talk) 03:35, 29 May 2012 (UTC)[reply]
It has more to do with the ratio of the wavelength to the dimensions of the sensor (typically, the human head). If by "low frequency" you mean something around 20 Hz, then the wavelength is about (300 m s-1) / (20 s-1) = 15 m. The diameter of the human head is about 2 orders of magnitude smaller so there isn't much phase difference between the ears at such low frequencies. Wikipedia's article on Sound localization is reasonably good. Short Brigade Harvester Boris (talk) 03:46, 29 May 2012 (UTC)[reply]
I don't think that is where he was going. Plasmic Physics (talk) 03:57, 29 May 2012 (UTC)[reply]
The OP has asked 2 things: a) what is the physics behind the fact that low frequency sounds travel farther than high frequencies, and b) why is it harder to tell the direction of lower frequency sounds.
With regard to the first question (a): PlasmicPhysics has it wrong, as he has confused ear sensitivity with attentuation of sound over distance. The attenuation of sound in air has two parts: 1) a drop in intensity to the third power of the distance, due to radiation in all directions (360 deg), so that a unit area covers a smaller angle as you move away; 2) attenuation above the third power due to the viscosity of air. This part increases approximately in proportion to the square of the frequency (if air comprised only of monatomic gas, it would be exactly proportional to frequency squared.) Viscosity is a form of frictional energy loss and is increased if you try to move the air faster. A method of calculating the attenuation in air is given in International Standard ISO 9613-1 (1993).
With regard to the second question (b): The brain appears to calculate the direction of a low frequncy sound from the phase angle beween the two ears. The phase angle is proportional to the ratio of wavelength to the distance between the ears. As frequency is reduced, the wavelength becomes so huge compared to the inter-ear distance that the phase angle becomes too small to sense. Form this, you would expect that direction accuity would be according to:
k d * (2 π f)
where k = a constant representing the efficiency of the brain calculation of phase, d = distance bewteen the ears (~180 mm), and f = frequency (Hz).
However, it is not that simple. Our perception of direction is often better than expected, still works to some degree for people deaf in one ear, and is far from fully understood. It may be that the brain has a way of utilising short-term phase memory (if the distance between the ears is insufficient, subconsciously move the head & compare) and wavefront bending around nearby objects, and/or some other trick(s). It has been shown by a number of researchers that if the head is prevented from moving (eg with clamps), a lot of our directional sensitivity is lost. An article and subsequent letters in the electronics journal Electronics World some years ago concerning artificial recreation of direction in headphones made this sort of trickery clearer. From memory the author was Ian Hickman, who tried using head position sensors connected to special electronics to alter the phase difference as the subject's head moved.
Ratbone58.170.163.145 (talk) 04:16, 29 May 2012 (UTC)[reply]
Neither the OP, not I have mentioned anything about ear sensitivity. Plasmic Physics (talk) 04:48, 29 May 2012 (UTC)[reply]
Well, you talked about noise. Thermal noise is what sets a limit on ear sensitivity (unless your ears are damaged.) Dispersed background noise, or noise above the thermal level, can set a practical limit on ear sensitivity. Noise has absolutely nothing to do with attenuation. Sound intensity will drop the same amount over a given distance whether acompanied by noise or not. Ratbone58.170.163.145 (talk) 05:02, 29 May 2012 (UTC)[reply]
I wasn't talking about ears. I was talking about how sound interferes with each other and scatters, and what role frequency plays in magnifying or diminishing the effects of scattering. Plasmic Physics (talk) 06:03, 29 May 2012 (UTC)[reply]
I can accept it, if I'm wrong, but I did not talk about ears. Plasmic Physics (talk) 06:06, 29 May 2012 (UTC)[reply]
For ears, I think the problem with determining direction is that there is very little difference in phase between the two because the frequency is so low. You would need two sites much further apart to tell the direction. I would be very curious to find more information about mechanically detecting low frequency sound, and especially about materials capable of blocking it (which might also be important for generating good directional sensors). The only thing I've noticed that sounds like it blocks low frequencies is a water-filled container (including parts of the body), but I've not found this discussed scientifically. Wnt (talk) 12:48, 29 May 2012 (UTC)[reply]
Thank you for the great answer. ```` — Preceding unsigned comment added by 65.92.6.118 (talk) 02:47, 30 May 2012 (UTC)[reply]

Refraction and entantiomers

Is it unreasonable for entantiomers to have differing refractive indices? Plasmic Physics (talk) 06:09, 29 May 2012 (UTC)[reply]

I think the answer depends on how you define what's different. Each entantiomer of a set has diferent refractive indices for each polarisation plane. For compound having two mirror forms, the second form will have the two values swapped over. Ratbone58.170.163.145 (talk) 07:06, 29 May 2012 (UTC)[reply]
Is there not some sort of standard, why do I regularly see only one RI listed for a substance? Does this mean that only one polarisation is shown? Plasmic Physics (talk) 07:59, 29 May 2012 (UTC)[reply]
As Ratbone pointed out, chiral materials have a different refractive index for right- and left-circularly polarized light. However, the average refractive index for both enantiomers is the same. Furthermore, the splitting of polarized light due to chirality is very small, typically a few parts per million (see [3]). This means that for practical purposes, there is no difference, especially when the refractive index is reported with only three or four significant digits. - Lindert (talk) 08:36, 29 May 2012 (UTC)[reply]
So, the standard is to show the RI for unpolarised light? Plasmic Physics (talk) 09:41, 29 May 2012 (UTC)[reply]
Assuming yes, what is the relationship between the RI for a mixture that has many diastereomers and the RI of each entatiomeric pair? Plasmic Physics (talk) 05:55, 31 May 2012 (UTC)[reply]

What evidence is there that DNA has a double helical structure in vivo?

After much furor at the talk page of Non-helical models of nucleic acid structure, I have been wondering what evidence is there that DNA takes on a double helical structure in vivo and not just in crystallographic determination, for example? This is in consideration of the fact that the cellular environment (especially of a nucleus) may be quite different to even the most thoughtfully prepared in vitro preparations of DNA that would allow structure determination of some kind (see macromolecular crowding). It is something I had never really thought about before until I read this article, which is not at all in a wiki style or holds a neutral viewpoint may I add, but now that I consider it I think it is a question worth asking. Any ideas? -Zynwyx (talk) 08:40, 29 May 2012 (UTC)[reply]

As mentioned on that same talk page, we do also have liquid-phase models of DNA structure produced using NMR. But then again, in those experiments the DNA is usually in relatively small fragments, and the rest of the solution is merely a salt buffer, rather than being filled with all the proteins, nucleotides, and other molecules you'll find in a nucleus. But for me, the best evidence for the double-helical structure is the corroboration of two types of studies. One would be ChIP-seq (working from fixed samples, it may as well be in vivo data) studies as well as older technologies such as DNA footprinting (entirely in vitro). The other would be NMR studies of DNA-binding proteins (or fragments thereof) and their accompanying DNA binding site (also in vitro). In many cases, you can actually show that the protein beautifully binds to a double-helix, right at the position predicted by both in vivo and in vitro methods.
Now, to play devil's advocate, it's also true that the selection of proteins for which there is an in vitro demonstration of binding to a double helix exactly as it should is precisely that: a selection. When a scientist fails to produce a comprehensible NMR spectrum, he won't publish it.
So yes, in a very strict sense, no one has ever shown direct evidence that the in vivo structure is a double helix, but I think the indirect evidence is basically overwhelming. I mean, shit, I can take a cell that is quite alive, fix it to preserve molecular structures, extract the chromatin, show that a particular protein is bound to a particular portion of the DNA in that chromatin, and then recreate the protein and the DNA sequence in vitro and show that they bind perfectly while the DNA is a double helix! I don't know how many proteins that has been demonstrated for, but it's more than a few. Someguy1221 (talk) 09:55, 29 May 2012 (UTC)[reply]
what about UV spectroscopy? don't the double helix and denatured forms supposed to give different absorption behaviors? Maybe one could look at the (almost) in-vivo spectrum and calculate the ratio of various different structures.Staticd (talk) 17:47, 29 May 2012 (UTC)[reply]
Thanks for the responses. I guess the fact that many DNA-binding proteins have a fit for the double-helix structure is a good indication that DNA is double helical in vivo. -Zynwyx (talk) 10:39, 30 May 2012 (UTC)[reply]
In vivo hydroxyl radical footprinting is being done [4], apparently without surprising results though I didn't chase down the Tullius group papers. The procedure randomly splits the DNA backbone, which shows which side of the strand is up against a protein and which is not. Since the strand is a double helix, that means that the pattern of breaks of each strand are regularly spaced. [5] Wnt (talk) 10:55, 31 May 2012 (UTC)[reply]

non-determinism and causality

Are non-determinism and causality mutually exclusive? I mean if we say determinism means that "the evolution and the present state of a system is the result of its initial state only." then non-determinism means that the present state of the system is not the result of its initial sate (at least not completely) Now if we agree that our system is non-deterministic, then we may ask "if the present state of the system is not only the result of its initial state, then what is the "other thing" that it is partially or completely the result of?" and because our answer can not involve an inside-the-system process (because we'll be back to determinism) it is either

1.An outside of the system process (which I think will either lead us back to determinism or an infinite loop, because then the final state will be determined by the initial state AND the outside-of-the-system process, and if that process is deterministic, then the whole process is deterministic, and if it's not, again we can ask the same question we asked about our first non-deterministic system, and we're in an infinite loop)
2.Nothing. in this case, our final state is at least partially caused by nothing, in other words it does not have a cause, so the statement "everything has a cause" will be wrong... so... am I wrong? where am I wrong in this reasoning?Because I always linked determinism and causality in my mind, and I didn't know where does that come from, and now I think I found a way to express it, and I wanna check if it's true?--Irrational number (talk) 11:20, 29 May 2012 (UTC)[reply]
You and I exist because our respective parents conceived. Everything we do (or don't do) is "caused" by something. ←Baseball Bugs What's up, Doc? carrots12:33, 29 May 2012 (UTC)[reply]
I don't know if this is relevant to the philosophical aspect here, but according to quantum physics there are things that are not caused by anything, but cause things to happen. Such as the radioactive decay of a particle. There is no timer, no cause, no determinism, no trigger, no hidden initial state that can be set or examined, no possible way to replay the experiment deterministically. It just happens when it happens to happen, and then can result in you digging a grave for a pet, or start a chain reaction that detonates a nuclear bomb. (Just rambling, but since you asked this on the science desk rather than at humanities...) 88.114.124.228 (talk) 16:49, 29 May 2012 (UTC)[reply]
What, as if the particle "decided" to do it? That's a pretty heavy implication. ←Baseball Bugs What's up, Doc? carrots22:50, 29 May 2012 (UTC)[reply]
I think that this is mostly a matter of definition. Causation tends to refer to relationships between individual events (the cause of death was a blow to the head). You can have a nondeterministic world where at least some events have identifiable causes (we seem to live in such a world) and I suppose there could be a deterministic world with no identifiable causes (because the state of everything at time t is affected by the state of everything at time t−1 in a way that's mathematically precise but impossible to actually calculate).
Another problem is that you can turn any nondeterministic theory into a deterministic one by adding some extra state to the world, and any deterministic world is unpredictable in practice because it's impossible to know the exact state at a given moment or compute the state at a later moment before that moment has arrived. So it's hard for me to accept that determinism or nondeterminism can be incompatible with anything; they seem to be the same thing. -- BenRG (talk) 20:55, 29 May 2012 (UTC)[reply]
One perhaps convenient definition of determinism vs. non-determinism is that in a totally deterministic worldview, all information about the future can be derived from information about the present. The convenience here is that it readily translates into quantum terms (which is the only place that determinism gets really weird anyway) — there are certain quantum interpretations which allow you to claim the world is deterministic (Einstein would have been cool with this) and those that would claim it is ultimately non-deterministic (e.g. the Bohr, Bell, and so on interpretations of QM). In the latter case, there simply is no ultimate repository of information until the wavefunction has collapsed and so on, and in some cases there is no way to know how that is going to go down. The relationship between that an causality strikes me as a complicated one. An unstable nucleus will decay at an unknown time, but we know why it decays — it is unstable. What we don't know know is why the decay event happened exactly then — there may be no reason, other than the fact that it is unstable and that's part of the definition of being unstable. Is that an acausal worldview? Bohr and Pauli thought it was, but I agree that it's largely a matter of definition. --Mr.98 (talk) 22:26, 29 May 2012 (UTC)[reply]
The problem with that definition is that it excludes cases where the future is determined, but not in any way that can be decided from information about the present (pure fatalism, for example). --Trovatore (talk) 00:10, 30 May 2012 (UTC)[reply]

What is the nature of life

If our genes are 99% like chimpanzees, what if anything, does the other per cent most closely resemble? ~ R.T.G 13:27, 29 May 2012 (UTC)[reply]

Yeah, and what about the other 1% in the chimp?165.212.189.187 (talk) 13:37, 29 May 2012 (UTC)[reply]
(short version of what I say below - traits that humans have that primates don't could occur in other species due to Convergent evolution)
Humans naturally have a rather anthropocentric view of the tree of life, and as such we think we're very different from animals like chimps. However on a grand scale primates show a lot less variation than a lot of other types of life we often group together with a single term, consider how varied beetles are, or how so may different types of life are called "bacteria". Indeed, in one perspective there's not much difference between all animals, we're all just differently shaped blobs of similarly constructed eukaryotes. On the billions of years timescale of the evolution of life the time that has been spent differentiating humans from other primates is small, so we only have a little DNA which has been naturally selected for humans' specific ecological niche.
As to what that 1% would resemble, if other members of homo were still around, we'd have stuff in common with them (as they'd be related). As to what currently existing non-primate animals will have similar genes, you'd have to look for occasions where animals have independently evolved characteristics that humans have (but other primates don't). It's not unknown for a gene to independently evolve two or more times - see the article on Convergent evolution. -- LukeSurl t c 16:55, 29 May 2012 (UTC)[reply]
The answer is complicated. First it is necessary to point out that the 99% figure only applies to the portion of DNA that codes for proteins, which is only about 1% of the total human DNA. The remaining DNA differs quite a bit more between humans and chimps, but it is hard to give precise numbers because it is hard to figure out how to do the comparison. In any case, many of the differences are random mutations with no describable structure, but lots of them arise from transposons such as Alu elements. In terms of the total genome, the thing that actually most strongly distinguishes humans is our Alus. Looie496 (talk) 17:17, 29 May 2012 (UTC)[reply]
"Take a chimpanzee. Stretch the body out, rework the musculature and bones a bit. Elongate the nose and numerous other fairly superficial changes (remove most of the hair). Radically increase the frontal lobe and rewire it considerably." --Mr.98 (talk) 12:35, 31 May 2012 (UTC)[reply]
Uh? I am very confused by the idea about this bit of DNA being only 1% Hope you still see the response! ~ R.T.G 19:51, 1 June 2012 (UTC)[reply]

How does the pressure-temperature diagram for refrigeration look like?

Most refrigeration cycles are described with the temperature vs entropy (T-E) diagram (fig 1).

Figure 1: Temperature–Entropy diagram.

However I would like to know how the diagram for pressure vs temperature looks like (fig 2).

I assume that in a fridge the tubes inside the food storage will contain gas which increase in temperature from the heat that leaks inside. The latter compression of the gas should cause an increase in temperature (and pressure) according to the ideal gas law (pV=nRT). The compressed gas would then inside the radiator tubing equalize with the surrounding ambient room temperature and thus decrease in temperature in order to loose energy to the environment. At the end of the radiator when the gas/liquid is expanded the temperature decreases (and pressure) and the cycle repeats.

Does the diagram below describe this correctly, and if not how should it look like?, except that after compression the media is likely in gas form in the beginning and not liquid as drawn.

Figure 2: Pressure-Temperature diagram. v2

Electron9 (talk) 16:45, 29 May 2012 (UTC)[reply]

Firstly, the refrigerant does not act as an ideal gas under the conditions that exist in refrigeration, secondly, your marked up graph shows compression moving from vapour to liquid. Compression must be entirely within a vapour region. Wickwack121.221.228.157 (talk) 02:24, 30 May 2012 (UTC)[reply]
Accurate now with compression completely in the gas phase and transitions between phases only when exchanging heat energy with the environment ? (how does the gas behave?) Electron9 (talk) 03:15, 30 May 2012 (UTC)[reply]
You have your marked up diagram nearly right now. Could you please repost your orignal mark-up, so that both are included? Otherwise the discussion will not make a lot of sense to anybody who finds this later on, either here or in the searchable archives. Expansion can cross the liquid/vapour line - show this by moving the cycle to the right. When refrigeration compression and expansion are plotted on a pressure-temperature diagram, the lines will be curved, not straight. However, I have never bothered to calculate the curvature, and like you, I have never seen it plotted on a P-T diagram in a textbook. When gasses are near the liquid-vapour region, they depart from the ideal gas laws. At low temperatures, pressure is lower than it would be for an ideal gas. Wickwack124.182.185.168 (talk) 03:46, 30 May 2012 (UTC)[reply]
Unfortunately the way commons work the old version can't be shown as an image with inline wikicode, but here is a link to version 0. I think the "can" makes it hard, as there will be multiple choice, expansion inside liquid or out to gas and that will contradict simplicity. Which way should the compression and expansion moves bend? Electron9 (talk) 04:41, 30 May 2012 (UTC)[reply]
Now version 2 shows the media expanding from liquid phase into gas phase (link to v1). I assume that's the most common case. But not necessarily the only possible one. Electron9 (talk) 04:55, 30 May 2012 (UTC)[reply]

Is it correct now?Electron9 (talk) 00:39, 31 May 2012 (UTC)[reply]

Physics/Math Theory ... Flat Flexible Object ... Volume Mass Force

Can a flat and flexible shape or object have Volume if a mass or force is applied to it? What I'm wondering is: if it doesn't have a top or lid on it, does it have Volume? I know that your thinking yes, it does!!!! But nobody's taught it to anyone, it's never been established. So, I want to know if it's been established or not. And, if not, would gravity affect if this theory is true or not. weight plus height plus mass equals Volume????--DAVINATOR8001 (talk) 19:28, 29 May 2012 (UTC)[reply]

What do you mean by flat ? If you mean a mathematical plane, with no thickness at all, then no, it wouldn't have any volume. But any real world object will have some thickness, even a piece of paper. So, yes, it does have a volume, and it can be calculated, given the thickness, length and width. StuRat (talk) 20:22, 29 May 2012 (UTC)[reply]
... and mass is connected with volume only through density (and weight is connected with mass only through gravity). Dbfirs
He's not talking about thickness or density. He's referring to this: Take a can. It has a volume. Cut off the top. The enclosed volume is still there, obviously, but it is not enclosed. Davinator is asking if that still counts as the volume of the can. With regards the flexible surface, if instead of starting with an enclosing object and cutting off one surface, he's asking about starting with a flat surface and denting it into something that could be a container but missing one surface. The answer to your question, Davin, is that a volume can only be defined for a closed region of space. Even if you cut the top off a can, you can still define the "volume of the can" as the region of space that would have been enclosed had the top still be there. Same as with bending a flat surface. But you always need to at least conceptually, if not physically, enclose a region of space. Someguy1221 (talk) 21:13, 30 May 2012 (UTC)[reply]
Yes, I suppose you could call that a "basin". This is a fairly important concept in geology, since, of course, lake basins don't have tops on them (unless you count an ice cap), but still have a volume they can hold before they overflow their banks. In general, however, the orientation of the object would control how much liquid it could hold. StuRat (talk) 22:47, 30 May 2012 (UTC)[reply]
If an aquarium holds 10 gallons of water, does half the aquarium hold 5 gallons of water? Each half, on its own, is not physically enclosed, except by an imaginary dividing plane, but that should suffice. Wnt (talk) 11:41, 31 May 2012 (UTC)[reply]
It would depend on how the aquarium is cut in half and how it is oriented. If you cut the top half off, then yes, if you cut it down the center, then no, but it could still hold some water if you tilted it properly. StuRat (talk) 05:03, 2 June 2012 (UTC)[reply]
This seems related to the concept of the convex hull. StuRat (talk) 05:03, 2 June 2012 (UTC)[reply]

Making Ammonium sulfate

I would like to make ammonium sulfate to make the soil in my garden more acidic- it is highly alkaline. I have ammonia cleaner -ammonium hydroxide- and some "soil sulfur," which I think is probably pure sulfur. Could they, or other household products, be combined to make the fertilizer? BeCritical 19:36, 29 May 2012 (UTC)[reply]

Urine is acidic. You could dilute it, say 10:1 with water, and spread it around the garden. It will take a long time to make a difference, but, on the plus side, it's free. StuRat (talk) 20:18, 29 May 2012 (UTC)[reply]
I'm dealing with extreme alkalinity. White alkali is collecting on the surface. Actually, I thought urine was alkaline... or salty. I'm no chemist. Anyway, I already use urine as much as I can... a lot. I just dumped gallons on the compost pile. I also use large amounts of coffee grounds. It's not enough. They make ammonium sulfate, but I thought maybe I could make my own. I would also like to know how much of the stuff like this it might take for this extreme condition? BeCritical 22:23, 29 May 2012 (UTC)[reply]
Have you had the soil tested? If so, please share the results so that we can better help. If not, why do you think you have highly alkaline soil? I highly suggest that you get a test if you haven't. They are very inexpensive, and sometimes free via an extension service. Other factors can cause white crusts to form on soil. 71.169.121.43 (talk) 22:38, 29 May 2012 (UTC)[reply]
Sulfur is not the same as sulfuric acid (which is what you'll need to make ammonium sulfate). Under standard conditions, sulfur reacts poorly with aqeous ammonia. At best you'll get small quantities of ammonium hydrosulfide. Vinegar is acidic, try that. Plasmic Physics (talk) 00:10, 30 May 2012 (UTC)[reply]
Reduce some vinegar by boiling untill it's very concentrated, let it cool, and add it to aqeous ammonia. This should make acetate of ammine, you can dry this at low temperatures to procure the powder, which you can then spread on your garden. Plasmic Physics (talk) 00:24, 30 May 2012 (UTC)[reply]

Cool interesting answer, thanks (: Sounds like it would work but wouldn't be practical for a whole garden... Just better to get the commercial stuff since the chemicals I have won't combine easily. But it would make a truly awesome school project for a kid huh? BeCritical 02:39, 30 May 2012 (UTC)[reply]

If want to prepare vitriolate of ammine, I suggest that you combine soda water with aqueous ammonia and natron (washing soda). This should give you aqueous carbonate of ammine, which can then be combined with vitriolate of magnesia (epsom salt) to produce your desired substance. All that is needed is to dry your substance. Plasmic Physics (talk) 04:56, 30 May 2012 (UTC)[reply]

The obvious place to look is Sulfuric acid#Manufacture (of course burning sulfur is dangerous). Icek (talk) 16:44, 30 May 2012 (UTC)[reply]

You could always bypass the calcification of brimstone - use vitriolic acid obtained as fresh battery acid. Plasmic Physics (talk) 19:44, 30 May 2012 (UTC)[reply]
It is much cheaper to just buy ammonium sulfate. Adding sulfur to soil is also a way to make the soil more acidic, by letting bacteria oxidise the sulfur to sulfate. You will need many kilograms of sulfur: do you have that much? Graeme Bartlett (talk) 22:00, 30 May 2012 (UTC)[reply]

What kind of latch ?

How is this type of latch called: X latch. Thanks for help! Grey Geezer 22:30, 29 May 2012 (UTC) — Preceding unsigned comment added by Grey Geezer (talkcontribs)

Maybe because its profile is vaguely X-shaped. Meanwhile, that website's purpose seems to be to pollute your screen with spam, so beware. ←Baseball Bugs What's up, Doc? carrots22:35, 29 May 2012 (UTC)[reply]
No, it is not called X latch. I called it "X" - like "unknown name". Grey Geezer 22:44, 29 May 2012 (UTC) — Preceding unsigned comment added by Grey Geezer (talkcontribs)
Can you find it on a non-spam site? ←Baseball Bugs What's up, Doc? carrots00:49, 30 May 2012 (UTC)[reply]
No. If I would have found it elswhere (I tried), I would know the name, right? Someone needs the name and uploaded it to the US-Imageshack site. Sorry. Grey Geezer 05:07, 30 May 2012 (UTC) — Preceding unsigned comment added by Grey Geezer (talkcontribs)
Here is the same image on imgur. The latch in question is called a draw latch. This particular one is a type with both a rubber arm and an over-center(?) pull. (I'm not certain of my vocabulary here. Perhaps it would be called a lever assisted rubber arm draw latch.) Note that in the photo, the cover or hood to be latched down is not in place, and so the latch is just "resting". Our latch (hardware) article does not offer very many illustrations and would benefit from a gallery of latch types. It does no more than admit to the existence of an animal called the "draw latch".-- 203.82.95.184 (talk) 06:29, 30 May 2012 (UTC)[reply]
This comes close. The correct (German) term is (=> GoogleImages) "Schaufelhalter" (shovel holder) but with "shovel holder" I could not find similar devices. Thanks everybody! Case closed. Grey Geezer 07:30, 30 May 2012 (UTC) — Preceding unsigned comment added by Grey Geezer (talkcontribs)

Color does not actually exists?

Is it true that color does not actually exists but is a figment of human anatomy (and a figment of living creatures' anatomy). That is to say in a universe devoid of life , there is no concept of color. 202.177.218.59 (talk) 23:09, 29 May 2012 (UTC)[reply]

I'd say yes, colour is just a construct facilitated by organisms to interpret different frequencies of light to receive information from the environment. Plasmic Physics (talk) 00:06, 30 May 2012 (UTC)[reply]
There is electromagnetic radiation and it comes in different frequencies. Our eyes and brains interpret a narrow (but rather useful) band of these frequencies as the qualia we call "color." The same can be said about sound (interpretation of pressure waves), taste and smell (interpretation of chemicals detected), and touch (interpretation of physical structure). All of our senses are equally "unreal" in that sense; it is what makes qualia interesting from a philosophical point of view. --Mr.98 (talk) 00:47, 30 May 2012 (UTC)[reply]
"Concept" is a human invention. In a universe devoid of life, there would be no concept of anything. ←Baseball Bugs What's up, Doc? carrots00:48, 30 May 2012 (UTC)[reply]
This is really a question of ontology. Color does indeed seem not to exist in a "physical" sense; as in there is no such thing as an "atom" of color. However I don't believe it can be said that color does NOT exist as a real property of matter. I've heard it described as a "meta-property" or an emergent property. The thing that really complicates the issue is that color is NOT just a property of the wavelength, for example in some optical issusions, one wavelength of light can appear as two completely different colors, but I think that just means that our perception of the "property" is more complicated, not that it doesn't exist at all. Vespine (talk) 01:12, 30 May 2012 (UTC)[reply]
The Church–Turing–Deutsch principle implies that qualia are just computational states of computer programs. Count Iblis (talk) 01:21, 30 May 2012 (UTC)[reply]
That seems to assume at least a couple of unstated premises. According to the article you linked, that principle "states that a universal computing device can simulate every physical process". To get your conclusion, you seem to need to know two extra things: That qualia are physical processes, and that being simulable by a universal computing device is the same as being a computational state. --Trovatore (talk) 02:27, 30 May 2012 (UTC)[reply]
To say that "color does not exist" is rather a bad way of putting it. It's the sort of thing that makes scientists sound really smart to some people, but it is not, strictly speaking, true. Light (and hence, color) is "seen" by humans and other living creatures as the most directly observable byproduct of electromagnetic radiation in the visible spectrum. Colors are not observed by every species (or even every individual) in the same way, so the experience of color is subjective, but that doesn't make it any less existent. Color exists as much as FM radio does. Evanh2008 (talk|contribs) 02:04, 30 May 2012 (UTC)[reply]

Color as a perception is a figment of human anatomy. Color as the wavelength of light (which causes the perception) is a property of light. There would be no concepts at all without life. BeCritical 02:58, 30 May 2012 (UTC)[reply]

David Eagleman makes the point (argues) that even different people percieve color (=> => interpret a "color" via the brain) differently. Your Blue is not my Blue. Grey Geezer 05:11, 30 May 2012 (UTC) — Preceding unsigned comment added by Grey Geezer (talkcontribs)
If a tree falls in a forest... --LukeSurl t c 23:14, 30 May 2012 (UTC)[reply]

FWIW, "perception of colour" is a pretty big topic. It varies from person to person (typical people can differentiate on the order of a million colours), animal to animal, by source of light etc. The exact same "colour" may appear bright red or bright green -- depending. This is far different from colour as bing a wavelength of light - which has a definable value. Collect (talk) 23:23, 30 May 2012 (UTC)[reply]

Colour results because of light and how that light reflects refracts absorbs on /off any given surface. It is the the structure of the surface of the object the light hits that will ultimately determine its colour. With out light there is no colour, and with out the surface there is no colour. For example a Bluebird's feathers are blue because of the way the feathers are placed and structured. Change that structure and the Bluebird will not be blue. Second, how light is perceived is as Collect says a huge field. But animals including human being's eyes are all structured differently. Even within human beings there are different eye structures. Male and female human beings have a different eye structure for example. Eye structure is the tool which "sees" colour but until the brain and eyes connect there is no perception of colour.(olive (talk) 20:04, 31 May 2012 (UTC))[reply]
A surface is usual, but the light source is part of it also. One can perceive color by looking directly at a light source, rather than a reflective surface. And the light reflected is also partially determined by the light source, that is what colors in what intensity are present in the source. A light source may have only one wavelength -as a laser- or many, and will usually favor certain colors, as the sun favors yellow. BeCritical 18:10, 1 June 2012 (UTC)[reply]
I guess I'm referring to the sun as light source without which there would be nothing visible coloured or otherwise and no other source of light would have been possible in our history, but my view comes out of colour theory for art so has a specific slant and or limitation, I suppose.(olive (talk) 19:09, 1 June 2012 (UTC))[reply]

Good or bad edit?

Can someone with a bit more familiarity with messier objects please check whether this was a good faith edit or vandalism and do what is appropriate? It's rare that you can't tell but here I have no idea—I could spend the time educating myself, but I'm sure someone with familiarity can do this much more economically.--Fuhghettaboutit (talk) 23:25, 29 May 2012 (UTC)[reply]

Bad. Only three in Puppis. (M46, M47 and M93 - I'll admit I had to look that up) FlowerpotmaN·(t) 23:31, 29 May 2012 (UTC)[reply]
Edit reverted FlowerpotmaN·(t) 23:32, 29 May 2012 (UTC)[reply]
Thanks Flowerpotman. I had a feeling it was vandalism, based on the magnitude of the change.--Fuhghettaboutit (talk) 23:55, 29 May 2012 (UTC)[reply]
No problem. I had to check how many there actually were in Puppis, but I did know there were 110 Messier objects and only 88 constellations, so, while they aren't evenly distributed.... well you can do the maths :) FlowerpotmaN·(t) 00:11, 30 May 2012 (UTC)[reply]
To be honest, I'd probably ABF simply because of the username... Brammers (talk/c) 23:07, 30 May 2012 (UTC)[reply]

May 30

Calories consumed during periods of intense activity

How many calories per day are typically consumed by a person (say an average-sized, fit guy) training for a marathon? Training for an Olympic swimming event? A triathlon or iron-man competition? I'm talking about high-intensity activities, where one isn't trying to gain weight (so, not weightlifting or its ilk). Now, roughly how many calories were consumed daily by Bobby Fischer and Boris Spassky during their chess competition? LadyofShalott 01:43, 30 May 2012 (UTC)[reply]

For athletes anywhere from 3500 Kcal/day up to 10,000 Kcal/day. Normal people will need about 2500 Kcal/day, one hour per day exercise will add 1000 Kcal to the energy needs and one hour per day is pretty much the lower limit of what you need to do to be able to perform at a decent amateur level. Michael Phelps is a example of someone near the upper limit of 10,000 Kcal/day, see here. Count Iblis (talk) 01:58, 30 May 2012 (UTC)[reply]
That's exactly what I needed to see - thanks! So I can rest assured that the crazy guy who claimed to me he'd eaten about 12-14,000 Kcal/day all his life (some of that time running about 4 miles/day) really is as crazy as I assumed. I asked about the chess players because he specifically mentioned that they'd comsumed such quantities during their tournament, and he'd eaten his highest amount when he was working on his invention. (I didn't learn what said invention was.) LadyofShalott 02:21, 30 May 2012 (UTC)[reply]
Four miles a day is not nearly enough to require that much food. Even the explorers who walked to the south pole from the coast of Antarctica were only eating 8,000 Kcal/day. There are certainly people who eat far more. There are documented individuals who consume in excess of 30,000 Kcal/day (the absorption of calories by the body is actually quite inefficient at that level), but all of them have one or more mental issues, and are quite obese. Someguy1221 (talk) 02:44, 30 May 2012 (UTC)[reply]
Thanks, this was not a fat person. You've both given me some good perspective on just how "off" the claim was. LadyofShalott 02:59, 30 May 2012 (UTC)[reply]

Goethian Optics

In his Theory of Colors, Goethe criticized Newton's theories of light and color. As far as I can tell, the theory he developed to explain the observations he made are nonsense. But that said, is there a way to make his observations compatible with Newton's ideas (in particular, light coming out of a prism, as in the picture in the article)? My guess is that a combination of Newton and the complexity of the human visual system can explain anything, but has a detailed analysis been performed? 65.92.6.118 (talk) 02:57, 30 May 2012 (UTC)[reply]

I believe Goethe's theory of light will always fail under modern (or even late 19th century) understanding of light due to his insistence on the physicality of darkness. We know well today that true darkness or blackness is the absence of detectable electromagnetic wave emissions off a surface, rather than "the polar opposite of light". And we further know that darkness does not interact with light to produce colors. What Goethe saw in his experiment was not an interaction of light and dark at the border of a beam projected through a prism, but the selective removal of wavelengths near the border. So once again, while his ideas were intriguing at the time, you have to forget the last 140 years of electrodynamics for them to be compatible with anything. As for Newton's own ideas, the article on Goethe's book doesn't go into detail on how Newton explained the physicality of the colors themselves - it merely mentions that he theorized they were all present in white light. In fact, Newton further theorized that light existed as rotating particles in a continuum of possible rotational frequencies. Each frequency corresponded to a distinct color, and the refractive index through a material boundary varied with rotational frequency. Given this fuller theory, it is impossible to merge Newton's and Goethe's theories without leaving out something essential. Someguy1221 (talk) 03:38, 30 May 2012 (UTC)[reply]
Oh, sorry, I thought you were asking to merge the theories, and here I reread and see that you were asking to explain his observations. But I did sort of answer that. But anyway, I think a fuller explanation only requires there things. The easiest experiment to explain is the creation of colors at the border of light and dark. As I said, this is due to a selective removal of wavelengths near the border. What you can imagine is that the light is passing through the prism as a column, and the colors are being separated into distinct columns. If the projection plane is too close to the prism, those columns overlap. In the center, all color-columns overlap, and you see white. But near the border, you are seeing just the edge of a single column, giving the appearance of red or violet. As you move the projection surface further away, the columns separate more, and the colors become more apparent. With regard the change in the color of light passing through a turbid medium, this may just be an effect of the absorption spectrum of the medium he was using (the article doesn't mention what its makeup was, merely that it was turbid). If that is the case, it simply means that his medium was absorbing the blue out of sunlight ever more at increasing concentrations, giving rise to a yellow or orange color. Now, with regard the blueness of and illuminated medium through which you are viewing darkness, there is an excellent experiment you can do outside your own home! Just look up at the sky during the day. It's blue. But you know that beyond that blue is actually darkness. What you are seeing is diffuse sky radiation. Basically, the blue wavelengths of light are being disproportionately scattered back toward your eye as sunlight passes through the atmosphere. This concept also applies to other media besides air, although you typically have to look through several meters to see a significant effect, and the color may vary with the material. Someguy1221 (talk) 03:52, 30 May 2012 (UTC)[reply]
"But near the border, you are seeing just the edge of a single column, giving the appearance of red or violet. As you move the projection surface further away, the columns separate more, and the colors become more apparent. "
Sorry, could you explain this a little more? Photos like the one in the dispersion (optics) seem to say that colours are always quite separated when they leave the prism. 65.92.7.168 (talk) 02:31, 31 May 2012 (UTC)[reply]
The first picture in the wiki dispersion article is not a photo, but a diagram. Whoever drew the diagram took an easy path and just drew specific colours, rather than the gradual variation that actually occurs. The second photo is of the spectrum of a compact flourescent lamp. You can be misled by such photos, because what such lamps emit is NOT white light, it is what looks like approximate white light to the human eye, and is not a continuous spectrum. It is a limitted number of specific colours. You cannot separate out what isn't there in the first place. To use a prism to show that white light comproses a continuous spread of wavelengths/colours, you must use a true white source, for which a back body radiator, such as the sun or an incandescent light, is a usable approximation. Ratbone124.182.150.22 (talk) 02:44, 31 May 2012 (UTC)[reply]
A wayyy better (re: more accurate) diagram can be seen about halfway down this page. Basically, in Newton's time there was no method for producing bright light in a laser-like beam. For color-splitting experiments, you instead put a circular aperture over a window and used the sun. What would come out is a relatively even column of approximately white light. For the image you pointed to, you can see that instead of a column of light entering the prism, it is a very thin beam. If you had access to a sufficiently thin beam, then yes, the colors would be separated immediately. If you like that concept, think of the column of light as a bunch of thin beams entering the prism at slightly different locations. When you're too close to the prism, the reds coming off of some beams overlap with the greens and blues coming off of some other beams. Someguy1221 (talk) 05:36, 31 May 2012 (UTC)[reply]
Not a wayyy better diagam at all. Not only does it show discrete colours where there should be a continuous spectrum, it shows no splitting at all within the prism. The separating out of colours in a prism is due to the fact that the refractive index of typical prism glass (or plastic etc) is a function of wavelength, ie colour. This means that some colour spreading occurs within the prism. At least our wiki articles get that right. And the 3rd diagram on Someguy's link is nonsense. Wickwack58.167.248.70 (talk) 11:34, 31 May 2012 (UTC)[reply]
Okay good, I thought the discrete color separation thing was fishy. But I still can't see why there would be white light coming out of the prism when the projection plans is close to the prism. 65.92.7.168 (talk) 15:47, 31 May 2012 (UTC)[reply]
The colours are only well separated when they leave the prism if the input to the prism is a narrow beam of light. Goethe illuminated the prism with a wider beam of light. With a wider beam, the colors come out at the same angles as with a narrow beam, but close to the prism the colors overlap. In the middle of the beam, green light from the middle of the input beam overlaps with red light from one side of the input beam and violet light from the other, and you get white light. Each color light propagates with a distinct angle, but light from different parts of the input beam start at different initial positions in the plane perpendicular to the beam's path. Because of this, different colours can overlap near the prism.--Srleffler (talk) 17:30, 31 May 2012 (UTC)[reply]
Perfect, thank you. 65.92.7.168 (talk) 18:33, 31 May 2012 (UTC)[reply]

2867 Šteins — common-noun nomenclature

Craters on 2867 Šteins are named for precious stones, and have been given the English-language names of these stones, e.g. Diamond, Opal, Jade. Are there other similar examples of solar system nomenclature where English-language common nouns have been officially assigned to features or objects?--Cam (talk) 04:52, 30 May 2012 (UTC)[reply]

Polar oceanic climate?

Since the talk page of Köppen climate classification is not very active, I am posting here. Where does Macquarie Island, which has average temperatures in the 1–9 °C range year-round, fall under the Köppen system? No type on that page seems to fit this model. GotR Talk 08:05, 30 May 2012 (UTC)[reply]

Why would the Tundra climate (ET) not fit the model? Condition is that the warmest month average is between 0 and 10 degrees Celsius. - Lindert (talk) 08:20, 30 May 2012 (UTC)[reply]
Most tundras regularly experience temperatures much colder than Macquarie's record low, and the island averages above freezing at night in its "winter". Köppen does distinguish between continental and oceanic subarctic, though. GotR Talk 09:08, 30 May 2012 (UTC)[reply]
It sounds like it's maritime temperate/oceanic to me. It narrowly misses the 10C threshold, but climate classification is far from an exact science. The key feature is that it has a very narrow temperature range, due to the moderating effects of being surrounded by ocean. That fits the spirit of maritime temperate, even if the temperatures don't quite fit the letter of the definition. --Tango (talk) 11:26, 30 May 2012 (UTC)[reply]
In strict terms it's a Koeppen ET climate. The Australian government calls it "extreme oceanic", which is a good description but as far as I know is not a formal classification. The overall character of the climate is more like the Cfc classification (which it only misses by a degree in one month) than ET. A good illustration of the arbitrary nature of climate classification. Short Brigade Harvester Boris (talk) 01:02, 31 May 2012 (UTC)[reply]

Birds that humans can ride?

There's the ostrich, emu and the mute swan. Are there any others? --95.148.107.232 (talk) 08:53, 30 May 2012 (UTC)[reply]

If a human tried to sit on a poor mute swan (only 12kg and about as tall as swans go), he would squash the life out of it. Wickwack60.230.210.162 (talk) 10:23, 30 May 2012 (UTC)[reply]
And I'd have serious doubts about riding emus too. (Have you seen one?) So, to the OP's question, no. HiLo48 (talk) 10:33, 30 May 2012 (UTC)[reply]
I saw a Chinese circus act in the late 1990's which featured a goose pulling a cart, in which sat a driver. I thought it very cruel when I saw it (and I still do).--TrogWoolley (talk) 11:05, 30 May 2012 (UTC)[reply]
Elephant bird? Extinct now, but alive until relatively recently in human history. --Mr.98 (talk) 12:37, 30 May 2012 (UTC)[reply]
However, riding an ostrich seems to be possible (see YouTube: Ostrich Riding); allthough why anyone would want to, remains unclear. Alansplodge (talk) 12:55, 30 May 2012 (UTC)[reply]

Since any answer is presumably restricted to ratites, I think the answer must be no.--Shantavira|feed me 16:05, 30 May 2012 (UTC)[reply]

The Cassowary is big enough but they lack the temperament to be messed with. SkyMachine (++) 21:55, 30 May 2012 (UTC)[reply]
Premature babies could be carried by a wide variety of birds. Or are they not human? 101.173.42.164 (talk) 10:05, 31 May 2012 (UTC)[reply]
Storks even. However it depends on how you define "ride". SkyMachine (++) 10:24, 31 May 2012 (UTC)[reply]
There are many stories besides the stork about people flying with the help birds. For example, The Little Prince has probably ridden many migrating birds at once; Baron Münchhausen recounts of the inhabitants of the Moon riding enormous vultures to war; some unknown guys in the József Attila poem “Indiában, hol éjjel a vadak” have tried to fly a chariot with eagles harnessed to it; Nils Holgersson has ridden on geese (though he was shrunk down at that time); Simonyi óbester has flown using birds in Móricz Zsigmond's novel Légy jó mindhalálig, etc. Apparently the trick in riding birds is either you need to be very light, or you need to harness more than one bird for a single bird isn't strong enough. Of course, in some of these stories it's questionable not only whether harnessing birds counts as “riding” them, but also whether the rider is human: the natives of the Moon in Baron Münchhausen's story are definitely not human. – b_jonas 12:35, 31 May 2012 (UTC)[reply]
Hmm wait, this is the science desk, not the humanities desk. – b_jonas 10:33, 1 June 2012 (UTC)[reply]

Woken up during non-REM sleep, i feel tired, why?, prevention?

When WAKED during NON-REM sleep, i feel tired in the rest of my wakefullness. in other words - someone wakes me in NRs? = will feel tired all day. (when waked in REM, feel active all day).

what is the name, mechanism, and possible prevention to this thing? thanks. — Preceding unsigned comment added by 79.177.180.234 (talk) 11:20, 30 May 2012 (UTC)[reply]

How do you know where you were in your sleep cycle? If you've been to a sleep clinic to have your sleep patterns analysed (which is really the only way you could know), then you should be asking the doctors there. If you haven't, then I would doubt the accuracy of your analysis. The article, sleep inertia, may be useful to you, but that shouldn't last more than a few minutes. --Tango (talk) 11:30, 30 May 2012 (UTC)[reply]

How to get hydrogen from electrolysis into a balloon?

Some time ago I tried to fill a small plastic bag with hydrogen obtained from electrolysis, but this didn't work. I put some sodium hydroxide in water and electrolysed that using a 40 volt DC power source. I measured the current, it was 0.1 amps. I had to make sure it didn't exceed 0.1 amps, to prevent the transformer from overheating, I did that by putting small amounts of sodium hydroxide in the water until the current was just below 0.1 amps.

The negative electrode was wound into a coil inside a small plastic cylinder. I attached this to the opening of a small plastic bag and then made sure it was sealed tight. The idea was that the hydrogen would collect into the bag. With 0.1 amps, this should happen at a rate of about 1 liter per day.

However, after one day, I didn't see any signs that the bag had inflated. So, what is going wrong here? Count Iblis (talk) 17:47, 30 May 2012 (UTC)[reply]

The "inside a small plastic cylinder" part confuses me. You need both electrodes to contact the water directly and have a direct electrical path from one to the other. If this plastic cylinder prevented that, this would be your problem.
Also, before trying to collect any gas, leave the collection balloon off, and see if any bubbles form (a few air bubbles will form in any case, but a continuous stream of rising bubbles means it's working). You could also use an electrical meter to see if any current is flowing (break the circuit and touch one end of the circuit with one probe and the other end with the other probe). StuRat (talk) 18:27, 30 May 2012 (UTC)[reply]
The best way I think would be to use some kind of a Hofmann voltameter, this way the oxygen and hydrogen is collected at the top of the seperate tubes. Once you see you have a decent amount of hydrogen collected, open the valve and collect it. - Lindert (talk) 18:45, 30 May 2012 (UTC)[reply]
StuRat, the cylinder is submerged in the water, and I can measure a current of 0.1 amp. Also I can see bubbles, the water changes color after a while (after one day it is a dark smelly mess). Somehow the gas isn't collecting in the bag. Lindert, yes, if I can make something similar to a voltameter, that would be an improvement. But it's not clear how I can make something like that with the stuff I have available, though. Count Iblis (talk) 19:54, 30 May 2012 (UTC)[reply]
Hydrogen diffuses (or maybe effuses?) quickly through a balloon wall, so my guess is that the same may be occurring with the plastic bag. Hydrogen molecules are very small and fast, so they diffuse through a thin solid like that more easily than other gasses do. Red Act (talk) 20:14, 30 May 2012 (UTC)[reply]
Very good point. When I did this in middle school, I used glass. Didn't collect much hydrogen, or very pure (due to the electrodes and electrolytes), but it was definitely there :) SemanticMantis (talk) 20:58, 30 May 2012 (UTC)[reply]
Plastic bags can work. My brothers made a "hydrogen bomb" out of a dozen garbage bags filled with hydrogen for one 4th of July. Made a nice deafening boom. However, maybe plastic bags are not all the same in this respect. StuRat (talk) 22:41, 30 May 2012 (UTC)[reply]
Proper electrolysis of water should generate hydrogen and oxygen - no place for a "dark smelly mess". Whatever your electrode is, it must be undergoing some electrochemical reaction that is replacing H+ as the acceptor for the electrons. Wnt (talk) 22:48, 30 May 2012 (UTC)[reply]
Oh, and StuRat, as this appears to be a real-life exercise, let's be clear that we're helping a valued colleague set up a basic science demonstration, not conspiring to produce an explosive device! Had the OP suggested any such intention I would rather strongly have discouraged people from directly involving themselves. Also note that the "deafening" in that statement is not a rhetorical flourish, but a real danger. Wnt (talk) 22:51, 30 May 2012 (UTC)[reply]
I can't imagine a faster way on to a government watch list than posting "Totally not making a bomb here guys!" :D RunningOnBrains(talk) 02:12, 31 May 2012 (UTC)[reply]
Agreed, "DON'T TRY THIS AT HOME, KIDS !" StuRat (talk) 23:07, 30 May 2012 (UTC)[reply]
In order to eliminate a leaking plastic bag from the list of possibilities, why not invert a glass jar over the electrode and see if you can collect gas inside it ? (A plastic jar would tend to float away and/or capsize.) StuRat (talk) 23:10, 30 May 2012 (UTC)[reply]

The dark smelly mess is undoubtably chlorine gas. Where is the water from? 203.27.72.5 (talk) 05:00, 31 May 2012 (UTC) Ok, I think I know what's going on here. The water you're using contains NaCl. You're also adding NaOH. The reaction scheme is as follows:[reply]
NaCl->Na++Cl-
2Cl-->Cl2+2e-
Cl2+H2->2HCl
HCl+NaOH->H2O+NaCl
The net effect is that you regenerate your reactants from your products and produce a little heat from the electrical energy. 203.27.72.5 (talk) 05:11, 31 May 2012 (UTC)[reply]

However, both chlorine and humid hydrogen chloride tends to eat through plastic bags and rubber balloons. Plasmic Physics (talk) 05:52, 31 May 2012 (UTC)[reply]
They also tend to stay in solution and react without ever getting to the plastic bag in any significant concentration. 203.27.72.5 (talk) 06:06, 31 May 2012 (UTC)[reply]
Also, you need to check your math regarding 0.1A gives 1 litre of gas/day.













203.27.72.5 (talk) 06:05, 31 May 2012 (UTC) My mistake. Should be x10s/C not x0.1C/s. Then the units work out and it's 23.9hours for 1 litre. 203.27.72.5 (talk) 06:37, 31 May 2012 (UTC)[reply]

I wouldn't say chlorine gas is undoubtedly a dark and smelly mass. Doing the electrolysis on a tiny scale as a kid, I don't remember it as dark at all, though sure, it could react. But the metal in an improvised electrode could also react. Look up the reduction potentials for various electrochemical half-reactions for various metals, chlorine, hydrogen. Oddly, I don't see a table here, but [6] covers some basics. Stuff that really hates electrons, like lithium or potassium ion, has a negative potential, hydrogen ion has a potential arbitrarily defined as zero, and stuff like chlorine gas that wants to be chloride has a positive potential. I'm not sure which metal would react and how to release a dark and smelly mass, or whether it's the metal itself or an oxide coating, etc. (I'm a bit rusty on this and perhaps someone will spot an error here...) Wnt (talk) 23:47, 31 May 2012 (UTC)[reply]
You're not going to get too many volatiles from metals, which would be needed to account for the smell. Chlorine from NaCl is the only candidate as far as I can see. Also, because the voltage supplied is 40V, and pretty much all of the ions in solution have an oxidation/reduction potential less than that, everything will react. I don't know what the electrodes are made out of, but assuming it's copper, it will enter solution as . That might account for some of the colour, but chlorine makes a dark green/yellow and it stinks. 101.171.213.82 (talk) 06:06, 1 June 2012 (UTC)[reply]
Metals often contain sulfur or arsenic, and they will result in smells (arsine hydrogen sulfide) when hydrogen reacts with them. With an inert electrode material such as platinum or gold you should not get that problem. Graeme Bartlett (talk) 06:41, 1 June 2012 (UTC)[reply]

Moon tilting

I have noticed that when waxing moon rises, the southeast part isn't lit. When moon sets, the northeast part is non-lit, so moon seems to tilt during the day. Why does this happen? I have noticed this from 60 N latitude. — Preceding unsigned comment added by 88.113.124.190 (talk) 22:16, 30 May 2012 (UTC)[reply]

During one night, the main effect you are seeing is the earth's rotation, relative to that the moon it self is roughly stationary. It does move across the sky a little bit, but only a tiny fraction of how much it appears to move due to the rotation of the earth. Draw a crescent moon on a piece of paper and pretend your head is the earth, keep the paper stationary on one side of your head and move your head through 180 degrees, now imagine you are standing on your nose and notice what the moon looks like from that perspective. Vespine (talk) 23:44, 30 May 2012 (UTC)[reply]

May 31

Animal bones

After an animal like cattle is slaughtered for its meat and skin, what happens to the bones? Can they be used in any useful way or are they just thrown out? ScienceApe (talk) 05:08, 31 May 2012 (UTC)[reply]

Cattle bones can be used to make bone char, or protein component can extracted for use in making gelatin. Some people also make jewelry out of it. I don't know though what percentage of cattle bones are actually put to each use, or whether some of it is simply disposed of. Someguy1221 (talk) 05:41, 31 May 2012 (UTC)[reply]
Some goes to making bone china, but not much these days I fancy. Richard Avery (talk) 06:50, 31 May 2012 (UTC)[reply]
You can also make Bone meal. As our article mentions, used as fertiliser and in the past as animal feed, but that largely went out of fashion after Bovine spongiform encephalopathy. Our article mentions Meat and bone meal which obviously also contains some bones has some other purposes. I may be mistaken, but my impression is nowadays very little is thrown away. (Our Environmental impact of meat production doesn't really discuss this.) Nil Einne (talk) 09:11, 31 May 2012 (UTC)[reply]
And, of course, beef stock, which is useful for many sorts of foods. 86.161.209.111 (talk) 13:18, 31 May 2012 (UTC)[reply]

For a skyscraper window washer, falling ....

How many floors (lets say at 3.6m or 12' high) would it take for a window washer falling from a skyscaper to reach terminal velocity so that any further floors dont make any difference? I ask because of this video [7] which speaks of a man falling 47 floors and surviving, even to [allegedly] a full recovery. Benyoch ...Don't panic! Don't panic!... (talk) 05:47, 31 May 2012 (UTC)[reply]

It would depend somewhat on the weight of the person and on the altitude. Atmosphere at higher altitude is thinner, and would give less resistance. The weight of the person matters because surface area (which catches the wind) is less relative to weight in a fat person than a thin one. There might also be other factors, such as variation in gravity at different points on the earth, and whether you are near the poll or equator. In addition, there are the physiological differences between people -basically how tough they are- that make a difference. Also the angle at which they hit the ground, and the tension in their muscles at the time (drunks survive accidents). So you can't pinpoint an exact number, but you might get fairly close. Someone else here can tell you about how fast he was going. He might have been cushioned by falling on top of his brother who fell with him. BeCritical 06:04, 31 May 2012 (UTC)[reply]
That was a fun simulation to write. Thanks for the question. Assuming that terminal velocity is 55m/s and that air resistance is proportional to the square of velocity, your acceleration becomes negligible after about 9 seconds, having fallen about 300 meters. Assuming 47 floors at 3.6m per floor, my simulation still shows him going 46m/s when he hits the ground after falling for a little over 6 seconds. If anyone asks why I bothered running a simulation, it's because it was easy to make and I don't remember calculus. Someguy1221 (talk) 06:09, 31 May 2012 (UTC)[reply]
Shape of the person and their clothing would also alter terminal velocity. 101.173.42.164 (talk) 10:08, 31 May 2012 (UTC)[reply]
In a real case scenario, other factors can help you survive. Hitting your leg on a marquise, falling on a tree, above a car, all these things might help you survive, even terminal speed. OsmanRF34 (talk) 17:03, 31 May 2012 (UTC)[reply]

In E = Mc2, Why c?

In Einstein’s equation, E=Mc2 , c is a constant representing the speed of light. It has always seemed odd to me that nature would use this specific value for both the speed of light and the immense value by which mass is multiplied to equal the amount of pure energy the mass contains.

As far as I know, neither nuclear fission nor fusion are fundamentally dependent upon the acceleration of mass to anything approaching c, let alone c2 (which, of course, would be impossible). Therefore, why does this specific constant pop up in an equation that calculates the amount of energy locked up in mass?

I am neither a physicist nor mathematician, so, if at all possible, could someone attempt a non-mathematical explanation? Honeyman2010 (talk) 06:55, 31 May 2012 (UTC)[reply]

Well, what distinguishes special relativity from Newtonian physics is the existence of a special speed, c, which shows up all over the place. Among other things it's the speed at which light propagates, but that's not fundamentally what it is, and it was probably a mistake to name it "the speed of light".
Because c is so fundamental it's often seen as an indication of the "correct" units to use for distance and time. E.g., if you use seconds for time then you should use light-seconds for distance, so that c = 1. Then the equation is just E = m and says that energy and mass are the same thing. -- BenRG (talk) 07:04, 31 May 2012 (UTC)[reply]
And if you don't like that explanation, it's also an unavoidable consequence a simple derivation of the equation, which you can see here. Someguy1221 (talk) 07:09, 31 May 2012 (UTC)[reply]
See also physical constant.--Shantavira|feed me 07:43, 31 May 2012 (UTC)[reply]
It's sort of the same thing as the way π shows up all over the place in many different sorts of maths. And how the golden equation exists. It seems to suggest something fundamental about reality, perhaps, but we start to leave science behind when we ask "why" in these cases. 86.161.209.111 (talk) 13:13, 31 May 2012 (UTC)[reply]
One does not necessarily leave science to fully understand how certain values relate to one another. The speed of light is not just the literal speed of light (it is that, of course), it's some sort of deep reflection of how physics is put together. We may yet understand that, fully within the confines of scientific thought, without recourse to mythology. --Mr.98 (talk) 13:41, 31 May 2012 (UTC)[reply]
Mythology? Since when is philosophy mythology? Since when are interesting mathematical ideas mythology? Are you under the impression that everything divides into 'science' or 'mythology'? I didn't think you were, but I don't see how else to parse your comment. 86.161.209.111 (talk) 17:02, 31 May 2012 (UTC)[reply]
To give a slightly different spin on what others have said, this equation does not appear in Newtonian mechanics. It only turns up when we start dealing with special relativity, a more refined theory of how motion works, that gives accurate answers in a wider variety of situations. The important assumption that appears in special relativity, but not Newtonian dynamics, is that if an object is moving at speed c according to one inertial observer (an observer travelling at a constant speed in a constant direction), then it will be moving at speed c according to any inertial observer. This assumption is needed for electromagnetism to work the same way according to any inertial observer, and we know it does due to various experiments like the famous Michelson-Morley experiment. The assumption turns out to have lots of interesting consequences, including your equation. Just to point out a possible misunderstanding in your question: it sounds like you are saying that it would be impossible for an object to accelerate to a speed of c^2 because that value would be greater than c. Instead, this is impossible because it doesn't make sense to interpret c^2 as a speed - c^2 is measured in m^2/s^2, while speed is measured in m/s. Saying an object moves at speed c^2 would be a bit like saying that a container holds 5 minutes of water. The units we measure speed in are arbitrary - we could use a system of units in which c=0.5, and the numerical value of c^2 in that system of units would be less than that of c. 130.88.73.65 (talk) 15:40, 31 May 2012 (UTC)[reply]
Expanding (perhaps a bit tangentially) on 130.88's final point, it's worth noting that there are a number of systems of so-called natural units used by physicists that assign simple values (usually exactly 1) to commonly-used physical constants. In Planck units, for instance, all of c, G (the gravitational constant), h-bar (the reduced Planck constant), and kB (the Boltzmann constant) are numerically equal to 1. TenOfAllTrades(talk) 15:54, 31 May 2012 (UTC)[reply]

You can also derive this starting from E = m, see here. To me this argument makes more sense as obviously c should be put equal to 1 in special relativity. There is no good reason within special relativity to have different units for distances and time intervals. Then to explain how to put back c when you are not allowed to use dimensional analysis, is what needs to be explained as I do on that page. Count Iblis (talk) 17:02, 31 May 2012 (UTC)[reply]

Every time that Count Iblis discusses his theory for measuring time- and space- in common units, I feel compelled to bring up several examples where time and space are not symmetric. You can use any unit you like, and you can set c == 1 (in units of some_spatial_interval_per_some_time_interval), but you can't ignore that it is not equal to one: it has units. If you ignore those units, then you break almost all of physics. Time and space are not "identical" types of physical quantities; at least, not for any reasonable definition of "physical." Time and space behave differently in almost every equation we use to describe physical phenomena; equivalently, almost all phenomena are described by equations that are not symmetric over an interchange between time- and space- variables. Nimur (talk) 01:09, 1 June 2012 (UTC)[reply]
I don't agree that considering space and time to have the same dimensions would break any laws of physics. It would merely break conventions. There is no problem whatsoever with doing physics without any units or dimensions, at least not theoretical physics. You can never do an experiment that could settle if two arbitrary quantities X and Y are dimensionally incompatable. That they are physically different is a vague ill defined qualification, you could also say the same thing for kinetic and potential energy. Count Iblis (talk) 01:31, 1 June 2012 (UTC)[reply]
And note that it's not my theory, it's the same position Michael Duff takes. In that article you see that Nimur's position above is similar to that of Okun, but he is wrong, c = 1 can be interpreted literally without it having units as Duff points out. Count Iblis (talk) 01:37, 1 June 2012 (UTC)[reply]

Reaction rate data for iodine recombination

The only source of reaction rate data for the reaction I + I + M → I2 + M, where M = I2 that I could find is that given in the NIST database ( http://kinetics.nist.gov/kinetics/ReactionSearch?r0=14362448&r1=14362448&r2=0&r3=0&r4=0&p0=7553562&p1=0&p2=0&p3=0&p4=0&expandResults=true& ), with gives:

A = 1.53; n = -5, and Ea = 0, for the arrhenious form k(T) = A Tn eEa/RT

Such a high value of n and low value of A seems very anomalous - n for this type of reaction is usually between 0 and -1, at least for elements in the P-Block of the periodic table. So I suspect it is in error. The trouble is, the reference given, Baulch, D.L.; Duxbury, J.; Grant, S.J.; Montague, D.C.; Title: Evaluated kinetic data for high temperature reactions. Volume 4 Homogeneous gas phase reactions of halogen- and cyanide- containing species (which is a book, not a journal as NIST states) is not available in any library I can access. Can anybody offer an alternate reference, or know where there is reliable data? I would like data for M = I too. Ratbone121.221.95.143 (talk) 08:23, 31 May 2012 (UTC)[reply]

Do we need to detox?

Do we accumulate toxins, that we need to get rid off? — Preceding unsigned comment added by OsmanRF34 (talkcontribs) 11:15, 31 May 2012 (UTC)[reply]

The human body has mechanisms to dispose of many naturally occurring toxins. However, depending on one's location, diet, etc., one may be exposed to high levels, or unusual kinds of toxins (e.g. due to water pollution). So there is no universal answer to the question. You could consult a doctor if you are concerned about a specific toxin. - Lindert (talk) 11:33, 31 May 2012 (UTC)[reply]
Indeed. Heavy metals are toxic and tend to accumulate. The most common problem is lead, which is ubiquitous in the environment due to decades of widespread use in paint, car batteries, gasolene, cable sheaths, all manner of things, even ladies' makeup. It appears that the body can get rid of lead, but the process is extremely slow, with a half life of many years. Wickwack58.167.248.70 (talk) 11:44, 31 May 2012 (UTC)[reply]
Actually the harmful lead in your body has a half-life of weeks. Only the lead in bone has a half-life of years, and that's generally not considered to be harmful. 203.27.72.5 (talk) 21:23, 31 May 2012 (UTC)[reply]
I am not concerned with toxins in my body, but with detox diets and detox products. Do we need any of them? — Preceding unsigned comment added by OsmanRF34 (talkcontribs) 11:37, 31 May 2012 (UTC)[reply]
I think Detoxification (alternative medicine) covers this well enough, what more do you need to know? Since this is the science desk, the fact they are not supported by science should answer your question Nil Einne (talk) 11:49, 31 May 2012 (UTC)[reply]
(ec) Wikipedia cannot be relied upon for medical advice, but the whole "detox" thing is, frankly, not medicine. See Detoxification (alternative medicine). LukeSurl t c 11:50, 31 May 2012 (UTC)[reply]
The general rule about detox diets is "They are a lot of nonsense". If a toxin has accumulated, it is becasue the body has not evolved a way of effectively excreting or metabolising it. In many cases, certain metals being an example, a way for the body to deal with it is simply not metabolically/chemically possible. This means that nothing you can eat or not eat can overcome the problem. However, some naturapathy remedies may (or may not) cure a problem, and be referred to practitioners as "detox" while not actually a toxin eliminator. Wickwack58.167.248.70 (talk) 11:54, 31 May 2012 (UTC)[reply]
You can eat chelating agents that will remove lead from your system. They will generally also remove essential minerals from your system and make you quite ill. 203.27.72.5 (talk) 21:26, 31 May 2012 (UTC)[reply]
Tim Minchin put it best. LukeSurl t c 11:58, 31 May 2012 (UTC)[reply]
This BBC story may be instructive; in it a scientist from the UK's Food Standards Agency explicitly calls detox diets "nonsense". I suppose if you consider money to be a toxin, these diets do seem very effective at purging you of that... -- Finlay McWalterTalk 13:16, 31 May 2012 (UTC)[reply]
But wouldn't be useful to eliminate, or try to eliminate, lead from our bodies? It tends naturally to bioaccumulate, and the amount of it in our diet and environment is increasing. Besides that, it doesn't seem to have any function in our organism. OsmanRF34 (talk) 14:14, 31 May 2012 (UTC)[reply]
Yes, it would indeed be useful. Unfortunately, being useful does not make it possible. Wickwack58.167.248.70 (talk) 14:58, 31 May 2012 (UTC)[reply]
It is possible, but the benefits do not outweight the side effects in all cases except acute lead poisoning. See chelation therapy. 203.27.72.5 (talk) 22:28, 31 May 2012 (UTC)[reply]
There is doubt over whether chelation therapy works; it probably doesn't, and may even make the problem worse. See "Medical management of lead exposure", page 5, in http://www.nhmrc.gov.au/_files_nhmrc/publications/attachments/gp2-lead-info-paper.pdf. Wickwack58.167.247.182 (talk) 02:14, 1 June 2012 (UTC)[reply]
Why do you think it probably doesn't work? That wasn't in your source. In fact, your source said "therapy...using metal-chelating agents...is strongly recommended for adults with acute lead poisoning". Also, I find it hard to reconcile chelation therapy not working with Harold McCluskey not dying from either radiation sickness or heavy metal poisoning. 203.27.72.5 (talk) 04:51, 1 June 2012 (UTC)[reply]
"the amount of it in our diet and environment is increasing" - this is completely false. Since leaded petrol and paint were phased out, the lead exposures of people in the developed world have dropped hugely. Obesity, smoking, and over-use of alcohol are the major external causes of ill health in the developed world, not lead. -- Finlay McWalterTalk 14:31, 31 May 2012 (UTC)[reply]
It is true that the amount of lead in the environment is not increasing. But it is also not true to assert that because lead has been eliminated from gasolene and paint then lead has ceased to be a problem. That's because a) lead in fuel was only one of several sources, and b) there's still a lot of lead-based house paint in situ, and people tend to sand it off and spread it around as part of refurbishment, dwellings are being constructed in former industrial areas contaminated with lead, and other reasons. Lead is a popular subject on Ref Desk, and a poster to a previous question cited a USA study that showed that US children have less lead now than in previous decades, but a sizable number are still over the US's arbitary and possibly rather high limit. In other countries, the picture isn't that good. I note that the USA has over several decades, had a much greater density of cars than other countries, and a greater percentage of light trucks was gasolene engines, making lead in gasolene more important. Here in Australia, children with lead over US limit gets reported in the media from time to time. It is true that lead, as generally found in body tissues, is not the obvious health impact of obesity, smoking, and excessive alcohol, but many authorities consider that lead has reduced the intelligence of the population slightly. Wickwack58.167.248.70 (talk) 15:29, 31 May 2012 (UTC)[reply]
I will post exactly what I posted last time; the idea that lead concentrations are not decreasing is absolute nonsense, and you are trying to invent doubt and controversy where none exists. Check these studies: (United States, further United States statistics, Australian study showing reduction since the mid-90s (page 5), albeit with smaller sample size, another Australian study showing significant reduction in over-exposure cases (page 13), the second half of that paper in case you were confused why it cut off in the middle). -RunningOnBrains(talk) 16:33, 31 May 2012 (UTC)[reply]
RunningOnBrians is not reading carefully before posting, and the links he's given do not refute what I said. I said lead is ubiquitous in the environment - the 2nd link he gave, an Australian Govt publication, says on page 2 "Despite removal of lead from petrol, lead is still ubiquitous in the environment and exposure continues." I have not said lead levels are not decreasing - I only said that despite remove lead from fuel and paint, the problem continues - a view which is supported by the links that RunningOBrains provided. RunningOnBrains has substantiated with linked references that there has been significant improvement in blood lead levels in the USA - which I have agreed with. But the USA is not the whole world, not even the whole developed world. In Australia, lead is mined or has been mined in several locations. As reported many many times in Australian media, what tends to happen is lead contamination occurs along the route and including the port city where the lead is transported for export. Every now and then the Authorities notice that children and animals are affected, and lean on the mining company to clean up their act, which (when they are being watched) they do. Then the Govt puts out a nice study showing the problem is fixed. Then it happens again. Mt Isa (refered to in one of ROB's links)and Esperance (refered to in one of my previous links: report http://www.esperanceport.com.au/downloads/inquiry/Leadissueupdate2.pdf) are examples of this. The big picture is that in individual areas, lead contamination goes up and goes down. Over all, the problem continues. This is NOT to suggest that mining is Australia's biggest lead problem. Its just an example of part of the problem. Lead contamination comes from many sources, as ROB's links confirm, particularly page 2 in the second link. Wickwack58.167.247.182 (talk) 01:56, 1 June 2012 (UTC)[reply]
"RunningOnBrians"? So who's "not reading carefully before posting"? :) ←Baseball Bugs What's up, Doc? carrots11:24, 1 June 2012 (UTC)[reply]
The first such dietary intervention I know of was by Aulus Cornelius Celsus, who wrote that "mallow or walnut juice rubbed up in wine" was an antidote for lead poisoning. [8] Apparently walnut [9] and Egyptian mallow [10] are fairly good at bioaccumulating lead, so I wouldn't entirely rule out the possibility of there being some sense to it, but there's certainly no modern evidence, and he didn't specify what compound might be involved. ;) Wnt (talk) 23:33, 31 May 2012 (UTC)[reply]
No, we do not need to detox in the same way as we need to breathe. It does have advantages, it may improve your health, but you're not required to detox. Only, if you've been exposed to something lethal, does it become neccessary. Just an idea. Plasmic Physics (talk) 23:52, 31 May 2012 (UTC)[reply]
Your idea is wrong Plasmic Physics. It does not have advantages, we are not getting contaminated by 'toxins', whatever that be, lead or not lead. 88.9.211.205 (talk) 00:08, 1 June 2012 (UTC)[reply]
Could toxins (at low levels) actually have health benefits? Some damage done to the body that then gets repaired may make the body stronger... Count Iblis (talk) 02:03, 1 June 2012 (UTC)[reply]
Yes they could. See Hormesis. 203.27.72.5 (talk) 04:57, 1 June 2012 (UTC)[reply]
What are you talking about? It does, and we could. Plasmic Physics (talk) 02:26, 1 June 2012 (UTC)[reply]

Which is More Buoyant?

Let's say I have two identical containers, each weighing zero. I fill the first one with a large amount of highly compressed air. The second one holds a vacuum.

Is it correct that the second one would be more buoyant than the first, because the air in the first weighs something, while the vacuum weighs zero?

Are there any other factors which would affect the buoyancy of either? Honeyman2010 (talk) 12:21, 31 May 2012 (UTC)[reply]

Yes, if the volume of the two containers is the same, the evacuated one and its (non)contents weigh less than the one with air. Immersed in some fluid (e.g. air or water) the evacuated one is more buoyant. Using this property to make a flying machine was proposed an impressively long time ago, as the vacuum airship. The hard thing is, of course, that in practice the pressure of the fluid surrounding a vacuum balloon will tend to crush it (and there's no pressure inside the evacuated vessel pushing back) - this reduces the volume of the container, so the airship doesn't fly. If you make the walls of the container strong enough to withstand the pressure (e.g. the 15 psi of air pressure) it has (now) to be made of something like steel - so the minor effective buoyancy of the vacuum is overwhelmed by the great weight of the steel vessel, and again the airship doesn't fly. That's why you've never seen a vacuum airship in the 350 years since it was proposed. Folks interested in the prospects of super-strong materials which may be made possible by nanotechnology think that some kind of diamond-fullerine-something stuff might be strong enough and light enough that it could hold its volume against air pressure and be so light that there would be a +ve net buoyancy. Theese "vacuum aerostats" are discussed a little in the novel The Diamond Age - but right now (as this is the science refdesk, not the science fiction refdesk) no material is (at least in a macroscopic scale) strong enough to do this, so vacuum aerostats don't exist. -- Finlay McWalterTalk 12:42, 31 May 2012 (UTC)[reply]
You can satisfy yourself that air has weight by a simple experiment with two balloons, (some sellotape), a stick, and some string. Take one piece of string and attach one end to the ceiling or similar, so that it hangs freely. Tie the other end of the string to the middle of the stick, adjusting to make it as equal as possible. Tie two shorter pieces of string, one to each end of the stick, again making it as balanced as possible. Blow up your balloons (stick a small piece of sellotape to each, to give a place to make a hole), tie their ends, use the strings on the ends of the stick to attach a balloon to each end. You now have a basic balance scale, which you should adjust until it is level. Once it is level, gently and carefully stick something sharp (like a needle) into one of the balloons (ideally into the sellotape) so that the air is released. As the air is released, that side of the balance should go up and the side that still has air should go down, showing that the air in the balloons has weight. 86.161.209.111 (talk) 13:03, 31 May 2012 (UTC)[reply]
sorry, fallacious. the inflated balloon also has buoyancy that cancels its extra mass. I know, I tried it once. see Archimedes' principle. Robinh (talk) 09:04, 1 June 2012 (UTC)[reply]
The buoyancy does not completely cancel out the added mass since the air inside the balloon is under pressure. Whether the added mass minus buoyancy can actually be measured by a such a crude scale is another matter. Someguy1221 (talk) 09:34, 1 June 2012 (UTC)[reply]

Transit of Mercury

There is an obvious problem with this article because there is a transit of Mercury in June 2012 and it is not listed. — Preceding unsigned comment added by 81.98.191.56 (talk) 12:32, 31 May 2012 (UTC)[reply]

anybody can edit Wikipedia..... 217.158.236.14 (talk) 12:35, 31 May 2012 (UTC)[reply]
Do you have a source? All sources that I can find say there is a transit of Venus, not Mercury. 137.108.145.21 (talk) 12:36, 31 May 2012 (UTC)[reply]
Including Transit_of_planet_Venus — Preceding unsigned comment added by 137.108.145.21 (talk) 12:37, 31 May 2012 (UTC)[reply]
The article is correct. It's consistent with NASA's list of past and future transits, which puts the next transit on May the 9th 2016, not 2012. -- Finlay McWalterTalk 13:01, 31 May 2012 (UTC)[reply]

Is Scientific Investigation Ever Irrelevant?

In a world of shrinking resources and growing poverty, is there a point at which scientific investigation becomes irrelevant, wasteful, and perhaps even immoral?

I love astronomy, cosmology, particle physics and more as much as any wannabe scientist. But I have trouble reconciling spending billions trying to find the Higgs boson while millions of our fellow human beings die every year from starvation and disease. Aside from generating Nobel Prizes, and providing a good living for a relatively tiny population of scientists, achievements such as finding the Higgs, or dark matter, or catching neutrinos or a million other such quests will not improve the life of anyone alive today or perhaps for many coming generations.

Shouldn't science encourage it's best and brightest minds to solve the fundamental problems of this world before tackling issues such as the shape of the Big Bang?Honeyman2010 (talk) 13:08, 31 May 2012 (UTC)[reply]

This is a common criticism of science funding but its validity ultimately hinges on being short-sighted. Only a narrow construct of "solv[ing] the fundamental problems of this world" includes only the adequate near-term production and distribution of food and medical supplies. Particle physics and other esoteric fields may not "solve the fundamental problems of this world" today, but the research findings may solve the future fundamental problems of this world. If there's one thing the history of science tells us it's that one cannot reliably predict the future value of basic science advances, but in aggregate it's typically a huge long-term payoff. — Scientizzle 13:20, 31 May 2012 (UTC)[reply]
(ec) Science is often driven by curiosity. Newton and Einstein did not have immediate economically profitable applications for their theories. It is impossible to know what amazing technologies will be made possible by fundamental research. It's like exploring unknown lands: You don't know what you will find, or what setbacks you will suffer. The expedition is expensive and you don't know if you'll ever earn it back. And yet without such curious people, we would not have many of the things we have. - Lindert (talk) 13:21, 31 May 2012 (UTC)[reply]
(WP:EC):Should we encourage scientists to study humanitarian problems? Sure, and we do. But that doesn't mean we can't also study other things. Part of why we value basic science is that we don't know what may or may not be useful in the future. Hardy is often quoted as saying "No one has yet discovered any warlike purpose to be served by the theory of numbers or relativity, and it seems very unlikely that anyone will do so for many years." In a sense, he was proud of the abstraction of his work, and if it didn't "do good", at least it would not be "bad", e.g. serve as a weapon. Of course, he was wrong. The theory of relativity is used in 'many' areas today, e.g. satellites and GPS. While these can be used as weapons, they also have saved many lives. Likewise, Hardy's number theory laid the foundation for modern cryptography, which is a similarly multi-use tool that can be used both to harm people and to help them. SemanticMantis (talk) 01:15, 1 June 2012 (UTC)[reply]
Wasn't the Manhattan Project based on relativistic science? And Robert McNamara certainly though there was a warlike application for number theory. 203.27.72.5 (talk) 21:02, 31 May 2012 (UTC)[reply]
On the Manhattan Project: not really. Nuclear fission is non-relativistic. You can use E=mc^2 as a shorthand for calculating the energy release per fission, but you can also do it classically as the repulsion of two positive masses (the energy is mostly kinetic). Relativity didn't play a big role in the Manhattan Project, though it does help understand some of the fundamental behaviors. Quantum theory was more important. --Mr.98 (talk) 00:33, 1 June 2012 (UTC)[reply]
I said "Of course, he was wrong" -- meaning that relativity and cryptography have been used for warlike purposes, even thought Hardy thought they wouldn't be in his near-future. (They have also been used for "good" purposes, as I alluded). SemanticMantis (talk) 01:15, 1 June 2012 (UTC)[reply]
I know. I was just pointing out how wrong he was...and conversely how right you were :). 101.171.213.74 (talk) 05:52, 1 June 2012 (UTC)[reply]
  • Lastly, changing what is studied comes down to scientific funding. There is no "science" that can "encourage" people to study a topic, there is only either funding available, or not. There are many politicians who campaign for less governmental science funding, and try to mock "useless" science being done today. Usually these same politicians also favor large defense spending, and don't mention humanitarian goals at all. So really, while there is a culture of science (or several cultures of several sciences), it's the power of the purse that ultimately determines what research gets done. (I'll let someone else cover how private/corporate funding of research effects what topics are studied;) SemanticMantis (talk) 13:28, 31 May 2012 (UTC)[reply]
Any time you start to think that too much is spent on science, just take a look at how much is spent on war. Science isn't the first place I'd start cutting. The NASA budget is $18 billion; the budget for the entire Department of Energy (which includes managing the nuclear stockpile aside from funding particle physics, etc.), is $30 billion. The entire National Science Foundation, which supports a vast array of basic university research, has a budget of $7 billion. The 2013 budget request for the Department of Defense is over $600 billion, and that's down from almost $700 billion in 2011. The Large Hadron Collider will cost something like $10 billion total, which is well under a billion a year for its construction. --Mr.98 (talk) 13:33, 31 May 2012 (UTC)[reply]
That's a fallacy. The Department of Defense also produces science. And without defense, you won't have any means to produce any science. OsmanRF34 (talk) 16:33, 31 May 2012 (UTC)[reply]
The DOD does spend about $70 billion for testing and developing new weapons systems (which involves some science but it's pretty narrow), and maybe $12 billion on more basic research. So that's not nothing, but it still means that about 90% of that spending is towards non-science. And the fallacy here is implying that cuts in the DOD budget would somehow mean you'd be "defense-less" in a way that would destroy the means to produce science. The vast majority of the current DOD budget is going towards fighting wars with countries or groups that in no way imperil US science. --Mr.98 (talk) 00:33, 1 June 2012 (UTC)[reply]
Historical segue: in the 1940s, in the middle of World War II, US policymakers were having just this discussion. Should science be funded with public funds, and if so, who should decide how those funds are distributed? Should they be applied to specific ends, or should they be also given out towards basic knowledge? A bill was introduced that would basically make a US science agency that would only work on applied research. Vannevar Bush opposed this, and connived FDR into letting him do a report on it, which became Science—The Endless Frontier, one of the classic defenses of basic research. Bush thought you had to let scientists dole out the money and that application couldn't be the reason for it; focused and applied research might work in places where you really knew what the outcome was supposed to be (e.g. the atomic bomb and radar, both of whom Bush were involved with), but it didn't guarantee you'd find unexpected things. The other problem with applied research is that you end up warping the science to fit the application — so we know a lot about any science plausibly related to military applications of the ocean, for example, but a whole lot less about things that aren't interesting to the military. You quickly end up with a somewhat "closed cycle" of knowledge that only feeds familiar, old problems.
Bush's scheme didn't totally play off but something similar was eventually created in 1950 as the National Science Foundation. To say the NSF is only about basic science isn't exactly right — they are supposed to give a lot of lip service to things with tangible benefits — but it's as close to that sort of thing as we have in the US. (All of this is described in brief [[11]].) --Mr.98 (talk) 13:52, 31 May 2012 (UTC)[reply]
We don't live in a world of growing poverty. That is simply untrue. Poverty has declined enormously in recent decades. You can see some statistics on this World Bank website. --Tango (talk) 15:29, 31 May 2012 (UTC)[reply]
Scientific progress is just about impossible to predict. I would guess that in the early 19th century, studying electricity must have seemed to many like an esoteric and pointless pursuit, yet within a few decades it led to all kinds of hugely important applications. 130.88.73.65 (talk) 16:00, 31 May 2012 (UTC)[reply]
You wouldn't know it from the mass media, but particle physics has actually seen a great drop-off in activity over the last few decades. The funding levels have gone way down, and I personally know a number of brilliant physicists who have migrated into other fields, including my own field of neuroscience. Looie496 (talk) 17:39, 31 May 2012 (UTC)[reply]
There's something very strange about the notion that basic research is important because some of it turns out to have applications—as though if we had the benefit of hindsight we would only want to do the parts with applications and not bother with the rest. Science itself is something that people want to do. It is its own point. You say that you love cosmology and particle physics. So do I. Their continued existence improves our quality of life, even if they never lead to improved ways of fueling and maintaining our 100-watt biological heat engines. -- BenRG (talk) 17:45, 31 May 2012 (UTC)[reply]
I agree, there is something strange about that argument. But it is a well-known argument that can be used to successfully defend the value of science to a more Utilitarian crowd. I also agree that certain fields like cosmology are, to some extent, "their own point". However, that is also weird, because it verges on equating certain types of science with certain types of art... which is an interesting discussion to have, but perhaps too subtle and nuanced for this venue. Basically, the discussion becomes no longer a topic for science per se, and instead a subject for philosophy (of science). SemanticMantis (talk) 18:09, 31 May 2012 (UTC)[reply]
"A more utilitarian crowd"—I argued that basic science is justified by the happiness it directly creates. I don't see how you can get more utilitarian than that. I think you mean a crowd of people who don't enjoy science. I'm not sure what those people do enjoy, but I'm pretty sure it's not art, since "basic art" gets even less funding than basic science. -- BenRG (talk) 20:33, 31 May 2012 (UTC)[reply]
I think the OP was assuming that you get some enjoyment out of knowning the results of some expensive scientific experiments, but he's just saying it's not the most utility you could be getting for your purchasing power. If you could chose between spending a billion dollars on food for the poor or a new particle accelerator, the food might give 1/10000 of the world's population a massive boost in their quality of life, whereas the accelerator might make 1/2 of the world's population experience a tiny joy and say "wow". 203.27.72.5 (talk) 21:14, 31 May 2012 (UTC)[reply]
I have trouble reconciling spending billions on popular music while millions of our fellow human beings die every year from starvation and disease.
Why don't you download it for free? — Preceding unsigned comment added by OsmanRF34 (talkcontribs) 18:09, 31 May 2012 (UTC)[reply]
Why exactly do you waste your money on internet access and electricity and buying a computer when that money could go towards feeding starving children in Africa? Certainly you do not need any of these things to survive. -RunningOnBrains(talk) 21:11, 31 May 2012 (UTC)[reply]
  • Insofar as human beings have any purpose beyond turning food into shit, one of those purposes should be expanding the collective knowledge of humanity. In that way, all basic science reseach is highly relevent, indeed I can think of no more relevent human than the expansion of collective knowledge. Everything else we do is turning food into shit. Every other form of life does that t0o. What makes us human is our ability to acquire and store knowledge as a species. If you take that away, you give up your humanity. --Jayron32 18:23, 31 May 2012 (UTC)[reply]
Imagine in the late 19th century, if the scientific community in general decided that the study of aeronautics was unnecessary. "Of what use is it for people to fly through the air? There are plenty of problems on the ground."    → Michael J    20:49, 31 May 2012 (UTC)[reply]
The sad part is that most economists know how to eliminate most poverty and ameliorate almost all of the rest, by returning to the high effective corporate tax rates of more prosperous times which would cause companies to invest in labor and production, creating consumer demand and growth, instead of paying the less risky small tax to bank their record profits, which leads to stagnation and widespread unemployment.
The problem is that raising corporate taxes is politically unpalatable to those with the most money (corporate officers) so with their powerful lobby virtually nobody knows about the macroeconomic effect of all corporations facing the same decision to either spend to hire and produce or bank profits. But it's the whole reason companies are charged "income" taxes on profits instead of receipts, so they can dodge taxes by growing, and it's how the U.S. paid off the World War II debt, which was three times today's relative to GDP -- by growing instead of running a surplus.
The way to solve this is to institute a mandatory corporate fiduciary duty to seek prosperous economic conditions, including advocacy of effective corporate income tax rates which make hoarding profits idle cost more than labor, production, and growth. How do we achieve that? I'll ask on the Humanities Desk. 71.212.251.217 (talk) 00:22, 1 June 2012 (UTC)[reply]
Interesting graphs, thanks. I think many of us agree that the issues brought up in this question--money and time spent on basic research as opposed to humanitarian goals -- are more governmental and philosophical in nature than scientific. SemanticMantis (talk) 01:15, 1 June 2012 (UTC)[reply]
but surely you can all agree that as the dollars dwindle less and less of them will go toward funding wacky futuristic pursuits and more will go toward, let's say...food for the day. Until the last dollar is spent entirely on food, etc. As the old native American proverb goes, only after the last fish has been caught will you realize that money cannot be eaten. As for the corporate tax discussion there is a delicate balance between taxing and having the corporation stay domesticated. If you tax them too much they can pickup and expatriate to another country.68.83.98.40 (talk) 12:15, 1 June 2012 (UTC)[reply]

Neutron Spin

What are the physical characteristics of a neutron which enable us to determine it has spin? — Preceding unsigned comment added by 174.62.167.57 (talk) 16:09, 31 May 2012 (UTC)[reply]

One thing that comes to mind is that neutron diffraction techniques allow the determination of magnetic ordering structure in a solid, because the spin on the neutrons interacts with the magnetic moments in the structure of the solid, whereas x-ray diffraction uses photons (which have no spin) and doesn't provide magnetic information. Brammers (talk/c) 16:58, 31 May 2012 (UTC)[reply]
Photons do have spin, actually. --Trovatore (talk) 20:50, 31 May 2012 (UTC)[reply]
Oops, apologies. So for those who are a bit hard of thinking tonight, how come the photon's spin doesn't result in magnetic peaks in x-ray diffraction patterns? Brammers (talk/c) 22:03, 31 May 2012 (UTC)[reply]
That's out of my depth, I'm afraid. I might speculate that it's because neutrons do have nonzero magnetic moment (presumably because it's made up of charged quarks, even though its overall charge is zero), whereas I think the magnetic moment of a photon is zero (but I'm not sure of that). --Trovatore (talk) 22:37, 31 May 2012 (UTC)[reply]
Our article, Spin (physics) has a section "Spin vector" which briefly describes particle-spin-detection methods. -RunningOnBrains(talk) 21:07, 31 May 2012 (UTC)[reply]
Nuclear magnetic resonance#Nuclear spin and magnets has a nice discussion of an effect of neutron spin. The result is that NMR susceptibility is isotope-specific, not element-specific. DMacks (talk) 04:31, 1 June 2012 (UTC)[reply]

How many fingers does a hand need?

So I was just playing Dungeon Siege III and in it, a goblin asks "How many fingers does a hand really need?" and it got me thinking. If you were to make a robot with hands, what is the minimum amount of digits it would require in order to do all of the physical tasks that humans are capable of? ScienceApe (talk) 20:26, 31 May 2012 (UTC)[reply]

Five. Consider one physical task that humans are capable of, that of holding four playing cards one each between the fingers. Stating your requirements in other ways would reduce the number of digits needed to three (ability to manipulate an object whilst always gripping it with two digits) or two (finger opposed to thumb enabling power and precision grips). Of course, if you want to redefine the digit sufficiently (make it very long and having a very great many degrees of freedom), you could get away with a single digit. --Tagishsimon (talk) 20:36, 31 May 2012 (UTC)[reply]
"need" is pretty dependent on purpose, as Tagishsimon demonstrates above. For a discussion about why hands are the way they are, see the hand article, specially the section on evolution. 88.9.211.205 (talk) 22:25, 31 May 2012 (UTC)[reply]
Depends on how much functionality you like in a hand. It's pretty easy to get an idea of what sorts of tasks you can do with two fingers, three, etc. Typing and the playing of musical instruments are two things that would likely be drastically affected by the number of fingers a theoretical person may have; of course, Steve Vai and Django Reinhardt would have two completely different answers to this question... Evanh2008 (talk|contribs) 22:30, 31 May 2012 (UTC)[reply]
Russ Conway was an example of a musician who was successful with fewer than the normal number of fingers. --ColinFine (talk) 23:52, 31 May 2012 (UTC)[reply]
OR: I'd go with three, two fingers and a thumb. Two would only give you stability along one axis, while more that three doesn't significantly improve things, at least not the way the human hand is laid out. As an added bonus, you could still give someone a recognizable finger. Clarityfiend (talk) 00:05, 1 June 2012 (UTC)[reply]
The original question was about robot manipulators. Some of the most powerful lifters I've ever seen are vacuum lifters - non-gripping armature attachments that use pneumatic pressure, rather than gripping contact force, to lift objects. Robots have all sorts of design-decision freedoms that biological systems do not. You can design a robot with zero or n fingers, or magnetic fingers, or sticky glue on their fingers; ... Here's a fantastic video of my former professor, lecturing on robot manipulator kinematics, and a transcript of the graduate-level material on robot armatures. The video shows many interesting robot armature manipulators and discusses how robot engineers model their physical capabilities theoretically, and translate those physics into computer program code. Nearly the full course is available online at Stanford Engineering Everywhere's "Artificial Intelligence - Intro to Robotics page. If I recall, lectures 4, 15, and 16 have extensive discussions about "finger"-like robots, and of course we worked on the Barrett WAM whole-arm manipulator robot in the more advanced experimental section (which is harder to post online!). Needless to say, it is much more difficult to program a robot with n extra fingers; simplistic robot manipulators use one-dimensional (one- or two-finger) grapplers or actuators. Nimur (talk) 00:25, 1 June 2012 (UTC)[reply]
Another robot interesting gripper that doesn't use fingers is this balloon filled with coffee grounds [12]. It uses suction to lock the balloon into a shape that grips via contact forces. SemanticMantis (talk) 02:00, 1 June 2012 (UTC)[reply]
A full set of fingers is useful when swimming. Alansplodge (talk) 17:18, 1 June 2012 (UTC)[reply]
Members of the pinniped family - "(from Latin pinna, wing or fin, and ped-, foot)" - seem to counter-indicate that assertion! They've convergently evolved toward a fin-like appendage; and though their limbs have similar anatomical structures to our fingers, they're not really useful for gripping. We have a whole article on this from the biological point of view: on opposition and apposition of fingers. Nimur (talk) 18:32, 1 June 2012 (UTC)[reply]

June 1

Trayvon and DXM (dextromethorphan)

Would the toxicology tests performed by Florida on Trayvon Martin have discovered DXM in his body (assuming he had DXM in his body)? --Kenatipo speak! 12:28, 1 June 2012 (UTC)[reply]

Is there any reason for asking? Dextromethorphan is very commonly used in cough syrup. It is also a hallucinogen in very high doses, but not very widely used since most people find its effects to be quite unpleasant. Looie496 (talk) 17:30, 1 June 2012 (UTC)[reply]
We have a recreational use of dextromethorphan article. Its popularity is rising (as a drug of abuse, not sure about the article's readership). DMacks (talk) 19:01, 1 June 2012 (UTC)[reply]
It appears that it isn't -RunningOnBrains(talk) 19:33, 1 June 2012 (UTC) [reply]
(EC) Even tho technically BLP doesn't really apply I'm reluctant to mention this but I get the feeling it will come up one way or the other. It seems there's a rumour flying around he was using DXM based on some Facebook posts he made a year ago. The infamous skittles and ice tea seem to tie in to this since it's suggested he was going to mix them with cough syrup to make Purple drank. Our article doesn't mention it but various sources like [13] [14] suggest skittles sometimes are used. The ice tea seems odd though, it would seem you want something fairly sweet and I know Americans food and drinks can often be fairly sweet but is ice tea really that sweet compared to soft drinks in the US? (Skittles is also slang for Coricidin pills.) Nil Einne (talk) 19:34, 1 June 2012 (UTC)[reply]
The idea that DXM would make someone violent to the extent that a busybody would shoot them is ridiculousness not far short of reefer madness. Incidentally it's not true that "most people" find its effects quite unpleasant. I actually rather recommend it. I also managed to persuade my father (who has a professional interest in neuroscience) to take it. Egg Centric 19:40, 1 June 2012 (UTC)[reply]

Why doesn't Canadian oil production match?

Hi there, maybe someone can help me with a mismatch I can't figure out. The Canadian Association of Petroleum Producers says that Canada produces about 2.55 million barrels of oil per day, but the CIA says it produces 3.483 million. Why don't they agree? Thanks, TastyCakes (talk) 14:31, 1 June 2012 (UTC)[reply]

First off, the 2.55 million figure is only for Western Canada. The whole of Canada produces 2.83 million bpd according to the 2010 CAPP report you linked. As far as the CIA factbook figure, it could be that they took a 2012 number and forgot to update the text which states that it is a 2010 number. It also could be a projection based upon 2010 figures. -RunningOnBrains(talk) 15:59, 1 June 2012 (UTC)[reply]
Ah yes, sorry 2.8 is correct. That still leaves a big gap though and I'm not sure I buy the idea that the CIA just mislabelled a 2012 estimate to a 2010 actual number. TastyCakes (talk) 17:48, 1 June 2012 (UTC)[reply]

Oiling the hair

Does Oiling the head-hair, for heaving "wet-look", has any biological advantages or disadvantages? thanks. — Preceding unsigned comment added by 79.179.151.250 (talk) 19:06, 1 June 2012 (UTC)[reply]

downcoast?

how is the time called when a mashine shut down and the components rotating till they stop? on sirens it is called coast down but how on machines?